gmat grammar – Online GMAT Prep Blog by PrepScholar /gmat/blog GMAT Prep Online Guides and Tips Thu, 22 Aug 2024 16:21:34 +0000 en-US hourly 1 https://wordpress.org/?v=4.8.3 6 GMAT Grammar Rules You Must Know /gmat/blog/gmat-grammar-rules/ Mon, 29 Jan 2018 14:00:10 +0000 http://gmat.psblogs.com/?p=2159 You may have heard rumors that you need to know a lot of grammar for the GMAT. Unfortunately, it’s true that you’ll see GMAT grammar questions; however, the test focuses on just a few main concepts you need to know. In this article, I’ll go over the six most important GMAT grammar rules, as well as … Continue reading "6 GMAT Grammar Rules You Must Know"

The post 6 GMAT Grammar Rules You Must Know appeared first on Online GMAT Prep Blog by PrepScholar.

]]>
grammar-389907_640

You may have heard rumors that you need to know a lot of grammar for the GMAT. Unfortunately, it’s true that you’ll see GMAT grammar questions; however, the test focuses on just a few main concepts you need to know.

In this article, I’ll go over the six most important GMAT grammar rules, as well as offer tips for how to learn them so you can ace the sentence correction questions on test day.

 

Overview: GMAT Grammar

Grammar is tested on the GMAT primarily in sentence correction questions, which make up about a third of the verbal section (36 questions in total).

In sentence correction questions, you will be shown a sentence with an underlined portion, and five answer choices that give options for how to replace the underlined portion. The first answer choice is always exactly the same as the underlined portion of the sentence.

Sentence correction questions on the GMAT will ask you to select the answer choice that will make the sentence 1) grammatically correct, 2) concise, and 3) logical. Most questions will test you on more than one of these factors at a time.

Keep in mind as you prepare for this section of the GMAT that GMAT grammar is based on specific rules. Even if a sentence would sound correct in normal daily conversation or writing, it might not be correct by GMAT standards, so it’s important to learn each of the grammar rules tested by the exam.

The good news is that GMAT grammar rules are also repetitive: as you practice, you’ll see the same rules being tested over and over again, which means that if you prepare well and thoroughly, you’ll know exactly what to expect on sentence correction questions.

 

Grammar is tested primarily on sentence corrections questions on the GMAT.
Grammar is tested primarily on sentence corrections questions on the GMAT.

 

6 Key GMAT Sentence Correction Rules

Let’s go over six of the most common GMAT grammar rules, alongside examples (authored by the Graduate Management Admissions Council, GMAC, which also writes the GMAT itself) from the GMATPrep Software.

 

#1: Dangling Modifiers

The GMAT loves to test you on modifiers, which are clauses, words, or phrases that describe other parts of a sentence. In the sentence, “My sister, the psychologist, is coming to visit,” for example, “the psychologist” modifies “my sister,” which is the subject of the sentence.

Dangling modifiers are the most common modifier-related errors on the GMAT. In a dangling modifier, the modifying phrase is misplaced, describing the incorrect word or phrase so that the sentence is illogical. Take a look at the following sentence and see if you can tell why it’s illogical.

A beautiful red-haired mermaid, the president thought Ariel was highly intelligent.

This sentence contains a dangling modifier: The president is likely not a beautiful red-haired mermaid (though that would be cool). The modifier “A beautiful red-haired mermaid” belongs next to the noun it modifies, which is Ariel. Let’s rewrite it to remove the dangling modifier:

The president thought Ariel, a beautiful red-haired mermaid, was highly intelligent.

Now let’s take a look at an official GMAT example.

dangling modifier 2

To spot dangling modifiers, look for introductory phrases followed by a comma. In this question, “architects and stonemasons” is the modifying phrase in question. The phrase needs to modify a noun, and one that is logical in the context of the sentence.

In answer choices B, D, and E, the modifying phrase is not following by a noun or noun phrase (answer choice E begins with a verb, ‘were,’ for example), so you can eliminate them.

In answer choice A, the phrase “architects and stonemasons” is modifying “huge palace and temple clusters.” It’s not logical for the palace and temple clusters to be architects and stonemasons, so you can eliminate this answer choice.

In answer choice C, “architects and stonemasons” is modifying “the Maya,” which makes sense in the context of the sentence. This is the correct answer.

 

#2: Proper Verb Tense

GMAT sentence correction questions often test students on proper verb tense usage. Remember that verb tenses (the basic ones are past, present, and future) give us information about when an action took place (or will take place, etc.). Verb tenses are generally supposed to remain consistent, unless there is a specific reason for the tense to shift within a sentence. For example:

By the time the police arrived, the robbers had fled.

Note the clue that the verb tense is supposed to change (in this case from past tense to past perfect): “By the time” lets us know that there was a sequence of events. In this sentence, the robbers fled before the police arrived, so that shift in time needs to be reflect by a shift in verb tense.

In the GMAT example below, the conditional construction of “if x happens, y will happen” requires the first half of the sentence to be in present tense and the second half of the sentence to be in the future tense. So we can immediately eliminate answer choices A and B.

verb tense shifts

Answer choices C and D are less concise and precise than answer choice E because of the extraneous clauses they include (‘as they already did’) and (‘as they have already’), making answer choice E the correct option.

 

#3: Illogical Comparison

In the sentence “Mark loves chips more than his mom,” what’s wrong?

Well, unless Mark really loves chips, it’s unlikely to be true. What the writer is probably trying to say is “Mark loves chips more than his mom does.” The original sentence is an example of an illogical comparison.

Illogical comparisons compare apples to oranges, meaning that the things being compared either don’t make sense in context or don’t match up with each other grammatically (nouns being compared to verbs, for example).

Clues of an illogical comparison question on the GMAT include phrasing like “more/less than,” “like/unlike,” and “as…as.”

In this example, you’re being asked to compare “a typical automobile loan” to something else (the word “unlike” clues you into this). So you need to look for 1) a noun, since “ban” is also a noun, and 2) a noun that’s logical in the sentence’s context.

illogical comparison 5

Answer choices B and D can’t be correct, then, because they compare the “automobile loan” (a noun) to prepositions (“with” and “for”).

Answer choices A and C are incorrect because, while they start with nouns (“lease-loan buyer(s)”), they are illogical; a lease-loan buyer can’t be compared to a loan itself.

Answer choice E is correct. It compares “a typical automobile loan” to “a lease-loan.”  

 

Illogical comparisons can be spotted using clue words like "like" and "unlike."
Illogical comparisons can be spotted using clue words like “like” and “unlike.”

 

 

#4: Pronoun Usage

GMAT sentence correction questions test you on proper pronoun usage. Correct pronouns match their antecedent (the word they stand in for). For example, in the sentence, “Sara took off her hat,” “Sara” is the antecedent for “her.” Plural pronouns (we, they, them, us) match with plural antecedents, and singular pronouns (I, he, she, her, him, it) match with singular antecedents.

Let’s look at one example of incorrect pronoun usage. What’s wrong with this sentence?

I never go to that restaurant because they have moldy cheese.

Who has moldy cheese? We can’t tell from the sentence. “They” is a plural pronoun, and its possible antecedents (“I” and “restaurant”) are both singular. We would rewrite it like this:

I never go to that restaurant because it has moldy cheese.

Now let’s take a look at a GMAT example of incorrect pronoun usage.

pronoun usage 2

In this example, the antecedent of “it” is “crab.” “They” and “their” (both plural pronouns) are incorrect pronouns because they don’t match with their antecedent (singular), so you can eliminate answer choices B and E. Answer choices A and D have no pronoun, so they are illogical (“because of living at great depths”—who/what is living at great depths?).

Answer choice C includes the correct, singular pronoun (“it”) and the appropriate verb (“lives”), so it’s the right option. Notice that it is also more concise than answer choices A and B, which is often a clue that an answer choice is a good one.

 

#5: Subject-Verb Agreement

GMAT sentence correction questions often include subject-verb agreement errors. The subject of a sentence must match with its verb; a singular subject has to accompany a singular verb, and a plural subject has to accompany a verb in the plural form.

A common trap you’ll see in subject-verb agreement questions is a prepositional phrase (a phrase that starts with a preposition and ends with a noun) that is wedged between the subject and the verb to confuse you.

Let’s look at an example of the “distracting extra phrase” trap:

This box of decorations belong upstairs.

The prepositional phrase in this sentence is “of decorations,” which you should ignore when figuring out the correct verb form to accompany the subject. “Box” is the subject, and it’s singular, so the verb form should be too; the singular present tense form of the verb “to belong” is “belongs,” not “belong,” so the sentence should read, “This box of decorations belongs upstairs.”

Now let’s go through a GMAT example of a subject-verb agreement error.

subject verb agreement 2

The subject you’re dealing with in this example is “cost” or “costs.” Its verb is “are” (before “prohibitive”), which is plural, so the subject has to be plural as well. This allows you to eliminate choices A and D right off the bat.

We’ll go over how to select the correct answer choice for this question in the next example.

 

#6: Idioms

An idiom, or idiomatic expression, is a common phrase in English, often involving prepositions. For example, one graduates “from” college, not “of” college. One “belongs to a club,” not “belongs with a club.”

GMAT sentence correction questions generally test more than one grammar error at once, and you’ll often see wrong answer choices that fix the original error but contain an incorrect idiom. Correct answer choices will always be grammatically and idiomatically sound.

Let’s look at an example:

idioms9

When two choices are being compared, “whether” is the idiomatically proper choice rather than “if.” That means that answer choices A and D can be eliminated immediately based on their incorrect use of “if.”

“Being” is redundant (and unnecessarily wordy) when used after “undergoing,” because “undergoing” already lets you know that the conversion is in process, so answer choices B and C can be eliminated as well.

Answer choice E is correct based on idiom usage, a lack of redundancy, and concision.

 

3 Tips for Learning GMAT Grammar Rules

There are a few key ways to target GMAT grammar to prep for the sentence correction questions. Let’s go over the three top tips for mastering the exam’s grammar questions, undertaking effective GMAT grammar practice, and learning GMAT sentence correction rules.

 

#1: Use Flashcards to Study Idioms

It’s important to learn about the most common idioms you’ll encounter on the test as part of your GMAT grammar practice routine so you can eliminate answer choices that use idioms incorrectly right away. Once you get familiar with them, answer options with incorrect idiomatic expressions speed up the process of elimination.

Particularly if you’re a non-native English speaker, learning idioms can be a tricky aspect of your GMAT grammar prep. Creating and studying with flashcards that include the meanings of idioms as well as examples of their proper usage can help you get comfortable with GMAT idioms. Check out our guide to GMAT idioms to familiarize yourself with the most common ones.

 

#2: Review the Underlying Grammar Concepts

Your GMAT grammar practice should include a review of the basics of grammar, including parts of speech, sentence structure, modifiers, verb tense, and pronoun usage. Princeton Review’s Grammar Smart includes an overview of many of the GMAT-relevant grammar rules, as well as drills to complete as part of your prep. I would recommend Grammar Smart as your primary GMAT grammar book over generic grammar guides because they aren’t likely to be specifically relevant to the GMAT.

 

#3: Learn How Grammar Rules Are Tested on the GMAT

When you practice with the GMATPrep Software or other study resources, target GMAT grammar rules by selecting sentence correction practice questions. As you learn to identify various question types, you’ll become more aware with the way each of the GMAT sentence correction rules are tested on the exam. This will help you learn how to approach each grammar concept as it is presented on the GMAT and how to work through each question type more efficiently, effectively, and accurately.

 

Regular practice is key to learning the GMAT grammar rules.
Regular practice is key to learning the GMAT grammar rules.

 

What’s Next?

For more information about the verbal section as a whole, check out our article on mastering all three GMAT verbal question types.

Our guide to the best GMAT verbal practice materials will help you prepare for the verbal section of the exam more effectively.

Trying to understand each of the GMAT sections in more depth? Our comprehensive overview of all four sections of the test will help you out.

The post 6 GMAT Grammar Rules You Must Know appeared first on Online GMAT Prep Blog by PrepScholar.

]]>
The Best GMAT Flashcards and How to Use Them /gmat/blog/best-gmat-flashcards/ Mon, 08 Jan 2018 13:00:29 +0000 http://gmat.psblogs.com/?p=1920 GMAT questions are complex, testing a lot of concepts at once, so you need to really know fundamentals like math formulas, idioms, and grammar rules. But what’s the best way to learn them? GMAT flashcards! They can help you practice these key quant and verbal concepts until they become second nature. In this article, I’ll talk about why flashcards are … Continue reading "The Best GMAT Flashcards and How to Use Them"

The post The Best GMAT Flashcards and How to Use Them appeared first on Online GMAT Prep Blog by PrepScholar.

]]>
night

GMAT questions are complex, testing a lot of concepts at once, so you need to really know fundamentals like math formulas, idioms, and grammar rules. But what’s the best way to learn them? GMAT flashcards! They can help you practice these key quant and verbal concepts until they become second nature.

In this article, I’ll talk about why flashcards are useful, how you should study with flashcards, whether or not you should make your own flashcards (spoiler alert: you should), and review some of the best GMAT flashcards available.

 

Why Should I Study With GMAT Flashcards?

For many test-takers, studying for the GMAT can seem like a long, difficult affair that involves many hours hunched over an in-depth review guide. Are simple tools like flashcards, then, even worthwhile for GMAT prep?

In a word: yes.

Flashcards are an incredibly effective way to prepare for both the GMAT quant and verbal sections. You can use GMAT quant flashcards to build your fluency with the formulas and concepts you’ll see on the GMAT quant section. Verbal flashcards, on the other hand, are a great way to improve your mastery of the idiomatic language and grammar skills.

Here are three reasons why flashcards are a great tool to help your GMAT prep:

#1: Flashcards Help You Confront the Material

Flashcards require you to actively engage with the material. For instance, rather than just trying to memorize a list of many different formulas you’ll use on the quant section, you’ll engage with each formula one by one. You’ll be forced to ask yourself whether or not you really know how to use the formula in a quick, concise way.

#2: Flashcards Are An Effective Study Tool

According to a study by Time, flashcards are one of the best tools you can use to master material. Every time you use a flash card, you’re training your brain to quickly and accurately recall information. As you practice with flashcards, you’ll be able to strengthen your knowledge and build your memory.

#3: Flashcards Help You Master Quant Fundamentals

Questions on the GMAT quant section often ask you to use more than one skill to get the right answer. Flashcards, on the other hand, only ask you to use one skill. In some ways that’s a drawback, but practicing each skill in isolation will help you achieve mastery of the fundamentals you need to succeed on GMAT quant. Then, when faced with a complex question you’ll be able to confidently combine those skills.

What Can I Study With GMAT Flashcards?

As I mentioned before, flashcards are a great tool to help you study for the GMAT, but they don’t have long, GMAT-style questions on them. Instead, they have bitesize chunks of information on them, such as an idiom or a particular math equation. You’ll use flashcards to build your fluency with these fundamental concepts that you need to master the longer, harder GMAT questions.

Most GMAT quant flashcards ask you about simple math concepts such as: “What is a polygon” or “Factor this equation.” These bite-sized math chunks are basically the fundamental high school math skills you’ll need to answer the more complicated math questions on the GMAT quant section.

Most GMAT flash cards for the verbal section focus on either idioms or grammar terms. Idiom flashcards ask you to explain the meaning of common idioms that you’ll see on the GMAT verbal section, while grammar flashcards ask you about grammar concepts that come up on sentence correction questions, such as identifying where to correctly place a comma in a sentence.

Again, GMAT flashcards don’t focus on GMAT-style questions. Rather, they focus on building the fundamental skills you need to answer GMAT-style questions.

 

Flashcards help break the larger concepts you need into small, bite-size pieces that are easy to master.
Flashcards help break the larger concepts you need into small, bite-size pieces that are easy to master.

 

How to Study With GMAT Flashcards

As with any study method, there are good and bad ways to practice. In this section, I’ll teach you the best way to study with GMAT flashcards to maximize your studying time.

The best way to study GMAT flash cards is to use the waterfall method. In the waterfall method, you work through smaller and smaller subsets of flashcards as you learn the different topics, then work back up through the cards once you’ve learned them all.

Let’s see how that works. Take your pile of flashcards.

startingstack

First, you’ll go through the cards individually. For each card that you can answer right easily, you’ll put it in the “Know it” pile on the left. For each card that you answer wrong or struggle with, you’ll put it in the “Struggled” pile on the right.

stack2

Once you’ve gone through all of your flashcards, pick up the “Struggled” pile. Go through this pile. For cards that you can answer, place them in a second “Know it” pile next to but not combined with the first “Know it” pile.

stack3

For cards that you can’t answer, make a new “Struggled” pile next to but not combined with the first “Struggled” pile. Repeat this process until your “Struggled” pile only has 4-5 cards. This process creates your “waterfall.”

stack4

After you’ve worked through all of your flashcards, you’re going to move back up the “waterfall.” Starting with the last “Struggled” pile, repeat these flashcards until you can answer every question in the pile.

stack5

Then, add the most recent (the rightmost) “Know it” pile. Go through these cards until you can get all of the answers right. If you miss any cards, shuffle the cards and start again. Continue this process with every stack of cards as you move back up the waterfall. Keep going until you can answer every card correctly.

stack6

The waterfall method works because you’re seeing the concepts that you struggle with the most often. You’ll study the cards you don’t understand more than you’ll study the concepts you’ve already mastered.

Eventually, you’ll have worked through your flashcards so often that you’ll know every concept.

 

Should I Make My Own GMAT Flashcards?

There are tons of free GMAT flash cards out there. Is it worthwhile to make your own?

Yes. The best flashcards are the flashcards you make yourself.

For one thing, writing down material is a great way to learn it. As you’re making your flashcards, you’ll be writing material, which will help you learn.

You can customize your flashcard set so that it contains more of the content you’re struggling with and less of the content you’ve already mastered. Building your flashcard deck with lots of cards on topics you struggle with will help you see those questions more often and lead to faster mastery.

When making your own flashcards, then, consider the topics you struggle with. Maybe you have trouble mastering the rules of probability. Maybe you really struggle with idioms. Build cards with those questions into your deck.

 

Writing your own flashcards helps you personalize the content so you see topics you struggle with more often.
Writing your own flashcards helps you personalize the content so you see topics you struggle with more often.

 

You’ll want to have two sides to each flashcard. On one side, you’ll give an example of a skill. For instance, you might want to write: “How do you find the area of a triangle?” On the second side, you’ll provide the correct answer to the question. You can also have a short question (such as, “Factor this equation”) on the first side, with the answer on the second side.

Ideally, you’ll make your flashcards on notecards so that you can use the waterfall method when you’re studying. You can purchase notecard decks or rings at any office supply store. You can also use an app to make your flashcards or create them in a program like Microsoft Word.

Keep your flashcards short and sweet. Flashcards aren’t the place to test your ability to solve long, multi-part questions. Each flashcard should only test one specific skill. Testing one specific skill means that you’ll have a targeted focus for each card and that you’ll be able to easily identify which skills you’re struggling with.

 

Best GMAT Flashcards Review

If you don’t want to make your own flashcards, or you’re looking to supplement your own flashcard deck with other flashcards to practice, there are a number of different options available around the web. There are free GMAT flashcards that you can download or use online, as well as flashcards you can purchase. In this section, I’ll review some of the best GMAT flashcards out there.

Beat the GMAT

  • Cost: Free with login
  • Format: PDF or app

Beat the GMAT’s downloadable flashcards cover all the major topics tested on the GMAT, as well as strategies for the verbal and quant sections. You can practice memorizing different formulas you’ll see on the quant section as well as applying the knowledge you’ve gained with short, one-step questions. These flashcards come in PDF form or through an app, so you can use them with the waterfall method or just pull out your phone if you don’t have a lot of space to work in.

 

GMAT Club

  • Cost: Free with login
  • Format: PDF

GMAT Club has an extensive array of flashcards that cover almost every concept you’ll see on the GMAT, from statistics to subject-verb agreement. This set is the most complete set of flashcards you’ll find on the GMAT. The questions are presented in easily digestible chunks which make the flashcards perfect for truly targeting your practice and mastering fundamentals one at a time. The PDF does have four different flash cards on one page, so you’ll have to spend some time with scissors getting them ready for use.

 

Magoosh

  • Cost: Free
  • Format: Online or in-app

Magoosh’s online GMAT flashcards are easy to use and get started with. You don’t have to have a login or download an app; you can simply visit the website to get started. The site breaks down flashcards by concept (e.g., algebra or number properties) and offers mixed concept review as well. The flashcards ask you questions about concepts more than they ask you to memorize formulas, such as “How do we add or subtract fractions?” If you’re looking for clear, quick answers, these cards might not be for you, as the explanations can be a bit wordy. Unfortunately, since you can only use these flashcards online, you can’t use the waterfall method with them.

 

Kaplan's GMAT flashcards ask questions about the structure of the test, as well as the content.
Kaplan’s GMAT flashcards ask questions about the structure of the test, as well as the content.

 

Kaplan

  • Cost: $12.70
  • Format: Paper flashcards

Kaplan’s GMAT flashcards are better for building your knowledge of the GMAT than for testing the actual content on the test. While there are cards that review grammar, math formulas, and idioms, a large percentage of the flashcards focus on the GMAT itself, asking questions about the different sections of the test or how much time you have for each section. The breakdown of the flashcards is 260 cards for the quant section, 190 cards for the verbal section, and 50 cards on the test format itself. If you’re not concerned about the mechanics of taking the test, I’d skip these cards. You can get better flashcards for free from Beat the GMAT or GMAT Club.

Ready4 GMAT (Formerly Prep4 GMAT)

  • Cost: Free with in-app purchases
  • Format: iOS/Android app

Ready4 GMAT (formerly Prep4 GMAT) is a great app that has a number of features, including practice questions, in-depth answer explanations, and personalized coaching and feedback. The app also has hundreds of flashcards that review important GMAT skills, like idioms and formulas. The app also contains flashcards that review vocabulary words you may see on the GMAT verbal section. While you can access the vocabulary flashcards and some introductory flash cards for quant and verbal for free, you have to subscribe to the app to use all of the features. A subscription is a one-time fee of $19.99 (not including tutoring).

The app has a sleek design and is packed with content, but can be buggy and slow. If you’re already thinking about purchasing the app, the flashcards are a nice bonus feature. If not, I’d use one of the other resources before trying this one. Unfortunately, since you can only access these flashcards via an app, you can’t use the waterfall method with them.

What’s Next?

Are you filled with flashcard card and wondering how to build flashcards into your GMAT study plan? Well, you’re in luck! Check out our total guide to crafting a GMAT study plan and learn about how many hours you should devote to studying with flashcards every week.

Wondering what other GMAT resources you can use to supercharge your prep? Don’t worry, we’ve done all the research for you. We’ve compiled guides on the best GMAT books, as well as the Best Online GMAT Resources (link coming soon!). Check out these detailed guides to find the resources that’ll work for you.

Looking for even more ways to achieve your GMAT goal score? Our guide to the top 10 GMAT strategies offers tips and strategies to help you ace your practice and the GMAT itself on test day.

The post The Best GMAT Flashcards and How to Use Them appeared first on Online GMAT Prep Blog by PrepScholar.

]]>
Which vs. That on the GMAT: What’s the Difference? /gmat/blog/which-vs-that-gmat/ Sun, 02 Jul 2017 00:00:26 +0000 http://gmat.psblogs.com/?p=2621 What’s the difference between “which” and “that”? When is it correct to use one or the other? What kinds of questions will you see about them on the GMAT? These are all great questions. The good news: that vs. which GMAT sentence correction questions aren’t as tough as they appear—even if you’re not a native … Continue reading "Which vs. That on the GMAT: What’s the Difference?"

The post Which vs. That on the GMAT: What’s the Difference? appeared first on Online GMAT Prep Blog by PrepScholar.

]]>
body_5Ws

What’s the difference between “which” and “that”? When is it correct to use one or the other? What kinds of questions will you see about them on the GMAT? These are all great questions. The good news: that vs. which GMAT sentence correction questions aren’t as tough as they appear—even if you’re not a native English speaker.

In this post, we’ll give you all the rules you need to know about which vs. that GMAT questions, an in-depth breakdown of all the kinds of sentence correction questions that test this concept, and some tips for easily distinguishing between the two pronouns (even in long, complex sentence constructions)! By the time we’re done, you’ll be able to nail any that vs. which GMAT question that comes your way.

 

What Do “That” and “Which” Mean? Why Are They Confusing?

“That” and “which”—along with “who,” “whose,” and “whom”—are in the group of words called relative pronouns. Relative pronouns are found at the beginning of a relative clause. Relative clauses function the same way adjective clauses do: they are used to define, identify, or give extra information about the noun that precedes them.

Here’s an example of a sentence with a relative clause:

Correct: Baklava, which many people love, can be tricky to make.

Here, “which” is the relative pronoun standing in for “baklava.” This sentence is correct as written, but many of you might be wondering: Why can’t it be written with “that?”

Incorrect: Baklava, that many people love, can be tricky to make.

And why does it need commas?

Incorrect: Baklava which many people love can be tricky to make.

Incorrect: Baklava that many people love can be tricky to make.

Relative clauses are a category of grammar that the GMAT loves to test you on; fortunately, the two rules below will clarify how to pick correctly between “that” vs. “which” every time.

 

baklava
Could somebody please make some baklava for me?!

 

Rule #1: That vs. Which = Restrictive vs. Nonrestrictive

Consider the following two examples:

Correct: The apples, which were on the counter, went bad. The strawberries, which were in the refrigerator, stayed fresh.

Also correct: The apples that were on the counter went bad, but the apples that were in the freezer stayed edible for months.

In the first example, the relative clause “which were on the counter” isn’t necessary to specify which apples you’re talking about. The subject of the sentence is clear and defined without the additional information. The reader can assume you mean all the apples present, and the same goes for the strawberries.

However, in the second example, the relative clause “that were on the counter” is necessary to specify which apples you’re talking about. Of all the apples, specifically the ones that were on the counter went bad. If you got rid of the “that” clause, the subject would be unclear—so the clause is essential to the meaning of the sentence. It’s not “by the way” information, like the “which” clauses in the first example are.

In official terms, the first example contains nonrestrictive or nonessential relative clauses, and the second example contains a restrictive or essential relative clause.

Restrictive clauses contain information that is necessary for identifying or specifying the nouns they modify. They begin with “that” (or “who,” if you’re referring to a person).

Nonrestrictive clauses contain information that is additional—it may be important stuff to know, but the nouns they’re modifying are clear and defined, even if you got rid of the clause. They begin with “which” (or “who,” if you’re referring to a person).

 

apple
Don’t let these delicious apples go bad!

 

Rule #2: Nonrestrictive Clauses Get Commas, Restrictive Clauses Don’t

Nonrestrictive clauses are “set off” by commas: one always comes before the clause, and one always comes after. Restrictive clauses do not get commas, as they are integral to the meaning and shouldn’t be “set off” from the rest of the sentence.

This rule is actually pretty intuitive; you’ve likely been doing it correctly without even thinking about it. To illustrate, here are some more examples of restrictive information and nonrestrictive information—and what happens to the meaning of a sentence when you confuse the two.

 

Restrictive Clauses (“That” Clauses) Have No Commas

Restrictive clauses, as essential parts of the sentence, get no commas (neither a comma before nor a comma after the clause). Here’s an example:

Correct: The gardens that were weeded carefully looked pristine and beautiful next to the ones that were neglected.

Here, of all the gardens, specifically the gardens that were weeded carefully are the ones that look pristine and beautiful. If we took out the relative clauses, the sentence would lose its meaning. Therefore the underlined clauses are restrictive and don’t get commas.

Incorrect: The gardens, which were weeded carefully, looked pristine and beautiful next to the ones, which were neglected.

This sentence is now nonsensical. What are “the ones?” This example illustrates why avoiding commas around restrictive clauses is vital for maintaining the meaning of the sentence.

Here’s another example of what happens when you try to make a restrictive clause nonrestrictive:

Correct: “To run outside in the summer heat, I need a shirt that is loose and comfortable.”

To run outside in the heat, I need specifically a loose and comfortable shirt—not all shirts.

Incorrect: “To run outside in the summer heat, I need a shirt, which is loose and comfortable.”

So…any given shirt is loose and comfortable? This sentence doesn’t make sense!

 

marathon runner
Gotta wear the right kind of shirt when you’re running!

 

Nonrestrictive Clauses (“Which”) Have Commas Before and After

You can think about nonrestrictive clauses as more or less a subcategory of parenthetical information (parts of a sentence that you could put in parentheses). A comma is needed before and after the clause, because it’s “interrupting” the flow of the sentence. If the clause were to be removed, the essential meaning of the noun it modifies would stay the same.

Correct: “When I go running, I wear my favorite shirt, which is really worn out.”

In this sentence, the fact that the writer’s favorite shirt is worn out doesn’t change the meaning of the noun “favorite shirt.” It’s simply a detail she decided to add—“favorite shirt” is already specific enough to be clear. Therefore, the clause is nonrestrictive and gets a comma before it, and it would have a comma afterward had the sentence not ended. The clause can be removed without making the noun it modifies suddenly unclear.

Look at what happens when you try to write it as a restrictive clause:

Incorrect: “When I go running, I wear my favorite shirt that is really worn out.”

Now, this sentence implies that of all of her favorite shirts, she specifically wears the one that is really worn out. But this implication is nonsensical, as when we say “favorite” we implicitly mean that there’s only one possible favorite.

 

shirtsonhangers
Which shirt is your favorite?

 

What about “Who”?

“Who” is used instead of both “that” and “which” when modifying people—so “who” can go with both restrictive and nonrestrictive clauses.

Here’s an example of a restrictive clause with “who”:

Correct: “Artists who sell thousands of paintings should make millions of dollars.”

In this sentence, the clause “who sell thousands of paintings” is a restrictive clause. It indicates that only those artists who are able to sell thousands of paintings should be paid millions of dollars.

If you tried to set this clause off with commas, it reads as such:

Incorrect: “Artists, who sell thousands of paintings, should make millions of dollars.”

Now, this sentence nonsensically suggests that all artists sell thousands of paintings, and all artists should make millions of dollars.

Here’s an example of a nonrestrictive clause with “who”:

Correct: My mother, who is an excellent seamstress, hems my jeans for me.

If we took out the relative clause, the subject of the sentence would still be clear: my mother. So this “who” clause is nonrestrictive and gets surrounded with commas.

It naturally follows that proper nouns (capital-letter nouns like names or official/branded places and things) can only use nonrestrictive relative clauses. This is because we already know exactly who or what we’re talking about—no additional specifying is needed to be clear. Here’s an example:

Correct: “Robert Frost, who wrote ‘The Road Not Taken,’ is one of the most famous American poets.”

Even if we removed “who wrote ‘The Road Not Taken’”, we’d still know that the subject of the sentence is Robert Frost.

Incorrect: “Robert Frost who wrote ‘The Road Not Taken’ is one of the most famous American poets.”

This probably sounds weird to you anyway, but if it doesn’t, all you need to do is memorize the rule: proper nouns, which include all names, always get nonrestrictive relative clauses.

 

brownow
Owls always say “who”… I’ll see myself out.

 

Rule #3: Pronouns Must Have a Clear Antecedent

Restrictive versus nonrestrictive aside, all pronouns have to have a clear and precise antecedent—meaning, there can’t be any ambiguity about what the pronoun is referring to.

Here’s an example:

Incorrect: Jake was surprised to find he had been voted team captain by his teammates who never had much self-confidence.

Who “who” is referring to in the above sentence is structurally unclear: is it Jake or the teammates who never had much self-confidence?

Presumably the author means that Jake didn’t have much self confidence, so let’s rewrite the sentence to make that apparent:

Correct: Jake, who had never had much self-confidence, was surprised to find he had been voted team captain by his teammates.

Jake is a proper noun, so the “who” clause is nonrestrictive.

If you see a sentence correction question on the GMAT in which it’s ambiguous what noun “that,” “who,” or “which” is standing in for, chances are that you need to reorganize the sentence completely. The right answer will never leave any doubt as to exactly what the relative pronoun is referring to.

Sometimes, the wrong answers will indicate the wrong antecedent for the pronoun, in which case you’ll still need to reorganize the sentence somehow.

 

Jake's been a great captain so far!
Jake’s been a great captain so far!

 

3 Which vs. That GMAT Examples

In this section, we’ll walk through one prime example of each kind of that vs. which GMAT sentence correction question.

 

Example 1: Restrictive Clauses

Musicians of the 17th century often enjoyed baroque style music that during the time was prominent primarily because of the works of J.S. Bach.

(A) music that during the time was prominent primarily because of the works of J.S. Bach
(B) music that was, during the time, prominent primarily because of J.S. Bach’s works
(C) music, which, because of the works of J.S. Bach, were primarily prominent during the time
(D) music, that was prominent primarily because of the works of J.S. Bach during the time
(E) music, which was prominent during the time primarily because of the works of J.S. Bach

First, let’s figure out what this question is testing. Look at the answer options: we have a few with “that” after the word “music,” a few with “which,” and varying comma placements throughout. So we can tell that this is a classic which vs. that GMAT question (i.e., restrictive vs. nonrestrictive relative clauses).

Now, we’re ready to solve it. To do so, we first have to identify the noun being modified, to see if the relative clause provides information that it is essential in specifying it.

The noun being modified by the relative clause in this question is the subject “baroque style music.” We don’t need any further specification: of all the music, we already know that talking about specifically baroque style music. Therefore, the clause after “music” should be nonrestrictive, so it should use “which” and have commas before and after it. Already, we can cross off (A), (B), and (D).

Of the two remaining, one contains a different mistake: (C) uses “were,” the past plural form of to be, even though the subject “baroque style music” is singular. (E) is correct as written—there’s no final comma after the nonrestrictive clause because the sentence ends, so that’s okay. (E) is the answer.

 

Why didn't Handel go shopping? Because he was too baroque!
Why didn’t Handel go shopping? Because he was too baroque!

 

Example 2: Nonrestrictive Clauses

Jean-Jacques Rousseau contended that man is good only when in “the state of nature” but is corrupted by society, that compels man to compare himself to others.

(A) man is good only when in “the state of nature” but is corrupted by society, that
(B) only man is good when in “the state of nature” but is corrupted by society, that
(C) man is good when in “the state of nature” but is corrupted only by society, that
(D) only man is good when in “the state of nature” but is corrupted by society, which
(E) man is good only when in “the state of nature” but is corrupted by society, which

Again, let’s take a look at the answer options to identify what concept(s) we’re being tested on. In the first part of the sentence, the placement of the word “only” moves around from option to option, so we’ll have to figure out where it should go to create the most clear sentence. In the second part of the sentence (after “society”), we have a relative clause that needs the correct pronoun, i.e., another classic “that” vs. “which” GMAT question.

For the “that” vs. “which” part, the first step is to identify the noun being modified, just like we did in the first question. The noun being modified by the relative clause is “society”—the author is stating that society compels man to compare himself to others.

To figure out if it’s restrictive or nonrestrictive, let’s ask ourselves: what would happen to the noun “society” if we removed “that compels man to compare himself to others.” Does the noun suddenly become less clear?

In this case, it doesn’t: That society “compels man to compare himself to others” is just additional info about all of society—it doesn’t indicate some specific type of society. Therefore, this clause should be nonrestrictive and use “which” after the comma. Already, we can eliminate (A), (B), and (C).

Let’s get back to the first part of the sentence now. (D) misplaces the “only” so that the sentence becomes nonsensical, so we can rule it out. (E) fixes both issues in the sentence and it’s the correct answer.

 

Society: it corrupts us.
Society: It corrupts us.

 

Example 3: Ambiguous Pronoun Use

You’ll often encounter sentence correction questions that “bury” which vs. that underneath more obvious errors. In fact, many of the wrong GMAT sentence correction answer options will fix the most obvious error in the sentence but contain other, more subtle errors (like ambiguous pronoun use). They do this on purpose: the incautious test-taker will pick the answer that corrects the main issue without double-checking that other errors aren’t present.

Here’s a classic example:

Proponents of artificial intelligence say they will be able to make computers that can understand English and other human languages, recognize objects, and reason as an expert does—computers that will be used to diagnose equipment breakdowns, deciding whether to authorize a loan, or other purposes such as these.

(A) as an expert does—computers that will be used to diagnose equipment breakdowns, deciding whether to authorize a loan, or other purposes such as these
(B) as an expert does, which may be used for purposes such as diagnosing equipment breakdowns or deciding whether to authorize a loan
(C) like an expert—computers that will be used for such purposes as diagnosing equipment breakdowns or deciding whether to authorize a loan
(D) like an expert, the use of which would be for purposes like the diagnosis of equipment breakdowns or the decision whether or not a loan should be authorized
(E) like an expert, to be used to diagnose equipment breakdowns, deciding whether to authorize a loan or not, or the like

As always, let’s look over the answer options. We have to decide between “as an expert does” and “like an expert” in the first part of the sentence, and then we have to decide on the best way to format the list of use cases for AI-enhanced computers. At first blush, there doesn’t appear to be any pronoun issues.

For the first part of the sentence, “as an expert does” is a perfectly fine substitute for “like an expert.” So we can’t eliminate any choices there. Let’s go ahead and tackle the second part of the sentence. As written, is contains an issue of parallelism—it should be “decide” and not “deciding.” So (A) is out.

(B) fixes the parallelism issue, but it creates another problem: the use of “which” is unclear. Computers should be the antecedent, but the placement suggests that “expert” could be the antecedent as well. Ambiguous pronoun use is always wrong, so (B) can be crossed off—along with (D) and (E), which suffer from the same issue.

(C) fixes the parallelism issue in the original version and avoids unclear pronoun use by putting “computers” back into the clause. (C) is correct.

 

Hopefully the computers don't enslave us all (well, more than they already have...)!
Hopefully the computers don’t enslave us all (well, more than they already have…)!

 

2 Tips for Which vs. That GMAT Questions

Below are the key tips for choosing the right answer on that vs. which GMAT questions.

 

Tip 1: Read The Sentence Without the Clause

To figure out if a clause is restrictive or not, ask yourself if the information is necessary to identify the noun. Does the meaning of the sentence significantly change when you state the sentence without the clause? Does the noun become unclear or undefined?

If yes, the clause is restrictive. Pick “that” and do not put commas around it.

If no, the clause is nonrestrictive. Pick “which” and put a comma before and after it (unless the sentence ends, then it just gets a comma before).

If you’re dealing with people, pick “who” either way, and use the above rules to decide if it gets set off by commas or not.

 

Tip 2: All or Some?

If you’re still stumped after removing the clause and seeing if the meaning of the sentence holds up, ask yourself if the noun that the clause is modifying is referring to all things of its type or some specific things of its type.

As you may have noticed, the third example GMAT question above happened to contain two restrictive clauses—one before and one after the em-dash. It was restrictive because the author was not talking about all computers, just some specific computers. Of all the computers in the world, the computers that specifically are artificially intelligent enough to understand languages and could do human-level recognition and reasoning tasks were the ones being discussed.

In the second example, the relative clause was modifying the noun “society.” There, the author was talking about all of society—not some specific subtype. This made the clause nonrestrictive.

So, if the clause indicates some specific subtype of the noun it’s modifying, the clause should likely be restrictive; if not, it should be nonrestrictive.

 

No matter what, the right answer in a sentence correction question won't have any grammar mistakes.
No matter what, the right answer in a sentence correction question won’t have any grammar mistakes.

 

What’s Next?

Our guide to the 11 best GMAT sentence correction strategies is a must read if you’re looking to ace these tricky question types.

Be sure to read through our guide to the six GMAT grammar rules you must know as well.

As always, the best way to get good at sentence correction questions is to practice, practice, practice.

Happy studying!

The post Which vs. That on the GMAT: What’s the Difference? appeared first on Online GMAT Prep Blog by PrepScholar.

]]>
93 GMAT Idioms and How to Use Them /gmat/blog/gmat-idioms-list/ Sat, 27 May 2017 02:00:20 +0000 http://gmat.psblogs.com/?p=2912 You might have seen GMAT idiom lists in prep materials or heard students talk about memorizing idioms for the GMAT. But how are idioms actually tested on the GMAT? How much do they really matter? And how can you familiarize yourself with them before the exam? In this article, I’ll go over what an idiom … Continue reading "93 GMAT Idioms and How to Use Them"

The post 93 GMAT Idioms and How to Use Them appeared first on Online GMAT Prep Blog by PrepScholar.

]]>
writer-1421099_640

You might have seen GMAT idiom lists in prep materials or heard students talk about memorizing idioms for the GMAT. But how are idioms actually tested on the GMAT? How much do they really matter? And how can you familiarize yourself with them before the exam?

In this article, I’ll go over what an idiom means in the context of the GMAT, how idioms are tested on the exam, and top tips for learning common idiomatic expressions, including how to create effective GMAT idiom flashcards. In addition, I’ll provide you with a GMAT idioms list of the 93 most common GMAT idiomatic expressions and examples of each one.

 

What Are GMAT Idioms?

An idiom is a common expression or grammatical structure in English. You might hear idioms described elsewhere as a “saying”—for example, “as easy as pie”—but on the GMAT, you won’t see these as often. Instead, you’ll encounter more everyday constructions and have to decide which one is correct in English.

For example, which one of these sentences is correctly phrased?

Henry graduated from college.

Henry graduated of college.

The first one is correct: You don’t graduate “of” school. You graduate “from” school. There’s no special grammatical reason that this is the case, other than that that’s simply the way we talk about graduation in English (using the preposition “from” rather than the preposition “of,” that is). This is an example of how you’ll encounter idioms on the GMAT: There’s no specific grammar rule to follow, but an idiomatic expression will need to be corrected simply because it’s not used in English.

So, how much do GMAT idioms matter for the test? This is a common question for students, and the answer isn’t black or white. If you’re a verbal whiz and used to reading high-quality materials, you probably won’t need to spend too much time on GMAT idioms, as you’re likely to know most of them already.

But if you’re a non-native English speaker or tend to have trouble with the verbal section, memorizing idioms might be more important for you to do. If you notice when you take practice tests early on in your prep that you’re missing incorrectly phrased idioms, you should devote additional study time to memorizing and practicing with common GMAT idioms.

No matter what, you should familiarize yourself with the most common idioms that you’re likely to see when you take the GMAT. As you go over the list later in this article, take note of any idioms that seem unfamiliar to you so you can memorize them. While you can’t memorize specific rules to prepare for idioms on the GMAT, the good news is that many of them do tend to show up on the exam over and over.

 

GMAT idioms are tested in sentence correction questions.
GMAT idioms are tested in sentence correction questions.

 

How Idioms Are Tested On the GMAT

We know now what a GMAT idiom is, but how will you be tested on them?

Idioms are tested in sentence correction questions in the GMAT verbal section. Sentence correction questions will show you a sentence that is partially underlined, asking you to choose between four rewritten replacements for the underlined portion (answer choices B through E) or leaving the sentence as is (answer choice A).

Sentence correction questions usually have more than one error. In idiom questions, often, there will be both an incorrectly used idiom in the given sentence and a broken grammar rule. One answer option may correct the sentence’s grammar, but still use the idiom incorrectly.

The right answer choice will correct both the grammar of the sentence and the incorrectly used idiom. For more information on the grammar rules you should know for GMAT sentence correction questions, go to our article on the 6 essential GMAT grammar rules.

So, what exactly do I mean by an “incorrectly used idiom?” There are three main errors you’re likely to encounter on GMAT idiom questions. Let’s walk through an example of each one.

 

#1: Preposition Usage

As in the “of/from” example in the prior section, many idiomatic expressions you see on the GMAT will be incorrect because they use the wrong preposition. Prepositions are words that show a relationship between a pronoun or noun and another part of a sentence. They include words like “of,” “to,” “for,” and “with,” as well as many words that indicate direction or location (“above,” “under,” and “below,” for example).

Idioms often involve the correct use of a specific preposition. On the GMAT, you might see those prepositions being used incorrectly.

Take a look at this example.

While larger banks can afford to maintain their own data-processing operations, many smaller regional and community banks are finding that the cost associated with upgrading data-processing equipment and with the development and maintenance of new products and technical staff are prohibitive.

  1. A) cost associated with
  2. B) costs associated with
  3. C) costs arising from
  4. D) cost of
  5. E) costs of

As in many sentence correction questions, there are two errors at play here: an idiomatic issue and a subject-verb agreement error. The subject of the verb “are” in the sentence is “cost(s).” Since “are” is the plural form of the verb “to be,” it needs to be preceded by a plural subject, so “costs” is correct rather than “cost.” This means that answer choices A and D can immediately be eliminated.

Now that we’ve narrowed the answer choices down to B, C, and E, we have to take a look at the idiomatic errors. “Associated with” is the correct idiom, while “arising from” and “of” don’t make sense in context, so answer choices C and E can be eliminated. Also, we need to match up the correct prepositions with one another: the “with” after “associated” needs to be followed by another “with.” “From” and “of” don’t match up with the first “with.”

One significant clue is the second use of “with” later in the sentence; “with” should be paired with “with,” rather than mixing prepositions (“of” and “with” or “from” and “with”). Thus, answer choice B is both grammatically and idiomatically correct.

 

#2: Correlatives

Correlatives are words that work together to serve a single function in a sentence, though they might be separated from each other within the sentence. You can think of these words as “married pairs.” Some examples are “both/and,” “either/or,” and “neither/nor.”

On the GMAT, many idiom errors involve correlatives being used incorrectly. Let’s take a look at an example.

A recording system was so secretly installed and operated in the Kennedy Oval Office that even Theodore C. Sorensen, the White House counsel, did not know it existed.

  1. A) A recording system was so secretly installed and operated in the Kennedy Oval Office that
  2. B) So secret was a recording system installation and operation in the Kennedy Oval Office
  3. C) It was so secret that a recording system was installed and operated in the Kennedy Oval Office
  4. D) A recording system that was so secretly installed and operated in the Kennedy Oval Office
  5. E) Installed and operated so secretly in the Kennedy Oval Office was a recording system that

The correct correlative in the context of this sentence is “so x that y” (see the idioms list below).

Answer choice B doesn’t pair “so” with “that,” and neither does answer choice D, so those two options can be immediately eliminated on the basis of idiomatic errors.

Answer choices C and E are illogically constructed.

In answer choice C, “it” is an ambiguous pronoun; we don’t know what “it” is referring to, so even though “so/that” is used in the sentence, the pronoun usage makes it the incorrect choice.

In answer choice E, the word order makes the sentence illogical; starting with “installed and operated” doesn’t make sense, as the subject of the sentence is “a recording system.”

Answer choice A, by contrast, starts with the correct subject (“a recording system”) and also includes the correct correlative construction (“so x that y”), so it’s the right option.

 

#3: Forms of Comparison

In English, there are certain rules about how we compare things to each other and which words we use to do so. For example, take a look at these three sentences. Which one is correct?

Between the two candidates, she was the better one.

Among the two candidates, she was the better one.

The first sentence is the correct one. When we compare two items (in this case, two candidates), we use the word “between” rather than “among.” So sentence #2 is incorrect because it uses “among” to refer to a comparison of two candidates.

The first sentence correctly uses “between” to compare two candidates, making it the correctly phrased one.

So now that we’ve selected “between” over “among” to compare two candidates, let’s go over another potential idiomatic error that you might encounter in comparison questions.

Between the two candidates, she was the better one.

Between the two candidates, she was the best one.

Superlatives (words like “best” and “worst”) are used when three or more items are compared. When there’s a comparison between only two items, “better” or “worse” would be correct. So sentence #1 is correct in this regard, using “better” to compare the two candidates.

Idiom errors on the GMAT may make comparisons using incorrect phrasing. Let’s look at an example.

The financial crash of October 1987 demonstrated that the world’s capital markets are integrated more closely than never before and events in one part of the global village may be transmitted to the rest of the village—almost instantaneously.

  1. A) integrated more closely than never before
  2. B) closely integrated more than ever before so
  3. C) more closely integrated as never before while
  4. D) more closely integrated than ever before and that
  5. E) more than ever before closely integrated as

We can tell this sentence might include an idiomatic comparison error because of the clue word “more.” The correct idiom to use in this comparison is “more x than ever,” not “more than never.” So we can eliminate answer choices A and C immediately.

Next, we can eliminate answer choices B and E, which include the correct phrasing (“more than ever”) but in the wrong order. “Closely integrated,” which is what’s happening “more than ever before,” comes before the idiom in answer choice B and after the idiom in answer choice E.

Answer choice C contains the correct idiomatic expression with the words in the correct order; “closely integrated” (x) comes after “more” and before “than ever.”

 

For sentences to be structured correctly, you have to compare like with like.
For sentences to be structured correctly, you have to compare like with like.

 

GMAT Idiom List: 93 Most Common GMAT Idioms

It’s important to know not only what idioms are likely to show up on the GMAT, but also how to use them correctly. Here’s a handy GMAT idioms list, containing 93 of the most common GMAT idioms and an example sentence for each one.

Idiom Example
A debate over A debate over the election results ensued.
A means to This job is only a means to an end for him.
A responsibility to The teacher has a responsibility to keep the children safe.
A result of The consequence is a result of your behavior.
Ability to He doesn’t have the ability to make better choices right now.
Act as While Dr. Martinez was at a conference, Dr. Johnson acted as the interim head surgeon.
Act like Stop acting like a child.
Agree on The team agreed on a solution.
Agree to When the police arrived, he agreed to a search.
Agree with He agreed with the sergeant’s findings.
Aid in The village needed aid in harvesting the season’s crops.
Allow for My work schedule doesn’t allow for many breaks.
Appeal to The contract appealed to my sense of order.
Are in danger of If Maria doesn’t pass the class, she is in danger of being placed on academic probation.
As/as I am as successful as he is.
Ask for My committee asked for my dissertation paperwork.
Associate with I associate the smell of cinnamon with the holiday season.
Attend to (someone) He attended to his sick wife.
Attribute x to y/x is attributed to y He attributes his depression to a death in the family.

Type 2 diabetes is sometimes attributed to obesity.

Base on We based our conclusions on the results of three peer-reviewed studies.
Believe x to be y The judge did not believe the evidence to be sufficient.
Between…and For her major, she has to choose between biology and chemistry.
Both x and y Both my advisor and my professor will be at the meeting.
Centers on The discussion centered on the film that we watched together.
Composed of The organization is composed of elected officials.
Concerned with Jason doesn’t concern himself with this matter.
Conform to He refuses to conform to social norms.
Consider x y (no “to be”) RIGHT: I consider Mark my friend.

WRONG: I consider Mark to be my friend.

Contend with My love for my daughter doesn’t content with my love for my son.
Created with The playwright wrote the play with the help of community members.
Credit to I credit my success to my mentors.
Decide on It’s difficult to decide on a course schedule when there are so many classes available.
Delighted to He told us that he would be delighted to assist us.
Depends on whether x The success of the contract depends on whether we are able to compromise or not.
Depicted as In the media, she was depicted as a villain.
Different from He is very different from his siblings.
Disclose to He disclosed his complicated history to the counselor.
Distinguish x from y It’s sometimes difficult to distinguish temporary grief from clinical depression.
Distinguish between x and y Toddlers start to learn to distinguish between right and wrong.
Doubt that I doubt that he’ll actually call back today.
Dwindle away We watched as our time dwindled away to nothing.
Either/or, neither/nor Either Dr. Smith or Dr. Cortes will lead the discussion.

Neither an apology nor a letter will be sufficient.

Elect as She was elected as the club’s vice president.
Elect to The voters elected him to office.
Enable to My health insurance enables me to choose the proper specialist.
Essential to Water is essential to life.
Estimate to be The average lifespan of a black lab is estimated to be 14 years.
Fascinated by My daughter was fascinated by the butterfly garden.
Further (for degree); Farther (for distance) She ran farther than I did.

We tried to contain the spill to avoid further destruction.

In contrast to In contrast to her earlier self, Mary is now outgoing and exuberant.
In contrast with x, y In contrast with Annie, David waits until the last minute to start writing his papers.
In the (morning, afternoon, evening) The plane is leaving in the morning.
Independent from Teenagers want to be independent from their parents.
Independent of That’s my personal opinion, independent of any media influence.
Indicate that The results indicated that our hypothesis was correct.
Indicate to My boss indicated to me that he was displeased.
Indifferent towards No matter how hard I try, I can’t be indifferent towards her.
Invest in They were both invested in the relationship, so they got married.
Invest with The judge is invested with the authority to approve of the will.
Just as…so too Just as exercise can lead to weight loss, so too can a healthy diet.
Modeled after We modeled the curriculum after Montessori educational materials.
Native of She is a native of Spain.
Native to This animal is native to Australia.
Necessary to CPS deemed it necessary to remove the child from the home.
Not only/but also The exam is not only difficult but is also lengthy.
Not so much/as I was not sleepy so much as exasperated.
Originate from The ritual originates from an ancient Nordic tradition.
Originate in The cancer originated in her lungs.
Originate with The idea originated with my brainstorming group.
Potential to Though he is struggling, he has the potential to improve.
Prefer x to y I prefer classic literature to contemporary works.
Prohibit x from y The school cannot prohibit the student from writing a controversial paper.
Range from x to y The kids range in age from 10 to 17.
Refer to I was referred to a specialist after my appointment.
Regard as We regard him as part of the family.
Require that x be y The group requires that participants be attentive.
Required from Our HR department doesn’t require that documentation from employees.
Required to He is required to submit to a drug test as a condition of his parole.
Responsible for He is not responsible for the child’s welfare.
Resulting in Our collective stress ultimately resulted in a family fight.
Rival in The massive tome rivaled War and Peace in length.
Sacrifice for Don’t sacrifice your personal life for your career.
Sacrifice to I sacrificed a great deal of time and energy to that demanding job.
Sequence of The sequence of events that unfolded shocked all of us.
So x as to be y He was so quiet as to be nearly unintelligible.
So x that y It was so cold that school was cancelled.
Speak about We need to speak about the incident.
Speak from I speak from experience when I say that you should hire an attorney when filing a claim.
Subscribe to I don’t subscribe to that theory.
Targeted…at All of our criticism was targeted at the committee head.
To sacrifice x for y We weren’t willing to sacrifice family time for church.
Unlike x, y Unlike the flu, allergies are not contagious.
X expected to y Taxes for the rich are expected to decrease next season.

 

 

3 Expert Tips for Learning GMAT Idioms

Now that you have an idea of how idioms are tested on the GMAT and what kinds of idioms will show up on the exam, let’s go over three expert tips on how you can best familiarize yourself with them.

 

#1: Make GMAT Idiom Flashcards

Using the above GMAT idioms list, make flashcards to use during your prep. As you complete practice tests, you should also note any additional idiomatic expressions you aren’t familiar with and add GMAT idiom flashcards that include them. You should incorporate flashcards into your regular GMAT idioms practice.

In addition to your own GMAT idiom list, you can also use pre-made GMAT idiom flashcards, such as those available on the Magoosh GMAT Idioms app. The app divides common GMAT idioms into “basic” (more common) and “advanced” (less common) categories and can be used to quiz yourself on the idioms you’re likely to see on the exam. Find out more about using apps to prepare for the GMAT here.

 

#2: Write Example Sentences

Another way to familiarize yourself with common GMAT idioms is to write sample sentences with them. It’s important to get to know the idioms you’re not used to in context, rather than simply memorizing them. It’s also important not to simply rely on your ear or intuition, as you might already be used to using certain idioms incorrectly. Using them correctly over and over will help you break that habit.

As part of a study session or a warm-up, select 10 idioms from the GMAT idioms list (preferably 10 that you’re not comfortable with already) and practice using them in sentences. As you do this, turns of phrase that might have sounded strange to you earlier will begin to sound correct, which will help you recognize idiomatic errors on the exam.

 

#3: Read—A Lot

To familiarize yourself with a wide variety of common English idioms (beyond those on your GMAT idiom list), you should read and digest complex materials such as The New York Times, The New Yorker, Science, Nature, and The Atlantic as a regular part of your GMAT prep.

This technique is especially important if you are a non-native English speaker, as like I said earlier, it’s important to familiarize yourself with idioms in context rather than to simply memorize a list. You’ll encounter many idioms as you read. Note any phrasing patterns that seem unfamiliar to you, and keep a log. Add them to your GMAT idioms list or to your flashcards so you can practice with them at a later time.

 

Reading and analyzing newspaper and magazine articles is a great way to familiarize yourself with common idioms.
Along with GMAT idiom flashcards, reading and analyzing newspaper and magazine articles is a great way to familiarize yourself with common idioms.

 

What’s Next?

Ready to delve into critical reasoning questions? Check out our tips on mastering critical reasoning questions on the GMAT verbal section.

Looking for GMAT verbal practice, including GMAT idioms practice? We’ve got you, with over 500 high-quality GMAT verbal practice questions.

Struggle with GMAT reading comprehension? Our guide to the nine best GMAT reading comprehension practice resources will help you improve.

The post 93 GMAT Idioms and How to Use Them appeared first on Online GMAT Prep Blog by PrepScholar.

]]>
8 Top Tips for GMAT Sentence Correction Questions /gmat/blog/gmat-sentence-correction-tips/ Fri, 05 May 2017 21:00:23 +0000 http://gmat.psblogs.com/?p=1241 What are GMAT sentence correction questions? What skills and concepts do they test? What are the most effective GMAT sentence correction tips? In this article, I’ll go over the format of sentence correction questions on the exam, the grammar concepts you’ll need to know to complete them, a few examples of the most common sentence correction error … Continue reading "8 Top Tips for GMAT Sentence Correction Questions"

The post 8 Top Tips for GMAT Sentence Correction Questions appeared first on Online GMAT Prep Blog by PrepScholar.

]]>
correcting-1870721_1280

What are GMAT sentence correction questions? What skills and concepts do they test? What are the most effective GMAT sentence correction tips?

In this article, I’ll go over the format of sentence correction questions on the exam, the grammar concepts you’ll need to know to complete them, a few examples of the most common sentence correction error types, and how best to prepare for these tricky questions before you take the GMAT.

 

The Basics: GMAT Sentence Correction Questions

Sentence correction questions are part of the verbal section of the GMAT. The verbal section contains 36 questions (including sentence correction, reading comprehension, and critical reasoning questions), and usually around 13 are sentence correction questions.

A sentence correction GMAT question will present you with a sentence, part or all of which will be underlined. Beneath the sentence will be five answer choices, each of which presents a different way to replace the underlined part of the sentence. The first of the answer choices will always be identical to the underlined portion. The next four will be different.

These questions test your ability to use English accurately and effectively, so the answer you select should be both grammatically correct and the most effective (concise, clear, and not awkwardly phrased) of the choices. The GMAT tests you in very specific ways, so it’s important to be familiar not only with grammar rules, but with the GMAT sentence correction rules in particular.

 

Identifying the error type is the first step to correcting a sentence on the GMAT.
Identifying the error type is the first step to correcting a sentence on the GMAT.

 

What Do GMAT Sentence Correction Questions Test?

If you’re worried about learning GMAT sentence correction rules, fear not: they only test a few basic grammar concepts and skills. On a given test, you’ll generally see multiple questions about each of the topics.

There are seven main error types you’ll find on GMAT sentence correction questions.

 

Subject-Verb Agreement 

GMAT sentence correction questions will require you to locate the subject of a sentence and its verb and to ensure that they match: a plural subject goes with the plural form of a verb, and a singular subject goes with the singular form of a verb.

In many sentences, the subject and its verb are right next to each other. In the sentence “He ran for mayor,” for example, the subject is ‘he’ and the verb, ‘ran.’ In sentence correction questions on the GMAT, locating the subject and its verb will be a bit trickier, usually because the subject and verb won’t be next to each other in a sentence. Take a look at this example, and try to find the main subject and verb:

Attempts to assuage his fear and soothe his conscience fails miserably, as we soon discover.

The subject of the verb ‘fails’ is ‘attempts.’ This is incorrect: ‘Attempts’ is a plural subject, while ‘fails’ is the singular form of the verb ‘to fail.’ (Remember that the singular form of most verbs ends in -s.) The corrected version would be:

Attempts to assuage his fear and soothe his conscience fail miserably, as we soon discover.

A GMAT sentence correction question will use the words in between the subject and the verb (in the case of this example, ‘to assuage his fear and soothe his conscience’) to confuse you or throw you off track. A good clue that a sentence correction question is asking you to correct a subject-verb agreement error is that the answer choices include both singular and plural forms of the same verb.

 

Reviewing parts of speech can help you prepare for subject-verb agreement questions.
Reviewing parts of speech can help you prepare for subject-verb agreement questions.

 

Verb Tense Accuracy/Consistency

In general, sentence correction questions will ask you to keep verb tense consistent through a sentence (past tense generally goes with past tense, for example). Sometimes, though, you might need to shift the verb tense if it’s specifically required by a sentence’s meaning, as in these examples:

We were cleaning up hurriedly when our parents suddenly arrive.

What’s wrong here? There was an ongoing activity happening in the past (‘were cleaning up’) that was interrupted, also in the past, by the parents’ arrival. In this case, the verb tense needs to shift because of a sudden change in activity. The correct version would be:

We were cleaning up hurriedly when our parents suddenly arrived.

To know how and when to shift the verb tense, look for clues related to time like ‘suddenly,’ ‘until,’ ‘already,’ and ‘by the time.’

An example of a correct verb tense shift:

Amal was always a good student, and he is still quite studious today.

In this case, the verb tense shifts from past to present to indicate what Amal is/was like at different points in time.

 

Being aware of how time functions in a sentence will help you with verb tense questions.
Being aware of how time functions in a sentence will help you with verb tense questions.

 

Idioms

Idioms are simply common ways of saying things in English, often having to do with prepositions. We say someone graduated ‘from’ college, for example, not ‘to’ college. John is worried about his finances because he doesn’t have ‘much’ money, not because he doesn’t have ‘many’ money. I have ‘less’ cake after the party, not ‘fewer’ cake.

Idioms come up frequently in GMAT sentence correction questions and can be especially difficult for non-native English speakers. It can be helpful to memorize a few of the most common ones that tend to crop up on the GMAT, but some students memorize lists of 200+ idioms. While this may make you feel better, it probably won’t help you improve your score too much: There will only be a few questions on the GMAT that require you to be familiar with idioms, and they’ll be different on every exam, so your prep time is likely better spent elsewhere. Just note the idiomatic expressions with which you’re not familiar as you practice, keep a running list, and become familiar with those.

 

shield-1519642_1280
Idioms are common ways of phrasing things in English.

 

Illogical Comparisons

Some GMAT sentence correction questions will ask you to correct an illogical comparison, or a sentence that compares two unlike things (for example, a noun to a verb, or two inappropriately matched nouns). For example, if you say:

He loves donuts more than me.

It’s actually quite a sad sentiment, as you’re saying that someone is fonder of donuts than they are of you. Obviously, what you really want to do is compare your love of donuts to his love of donuts. Correct versions of the sentences would read, “He loves donuts more than I love donuts,” or “He loves donuts more than I do.”

Illogical comparison questions are quite easy to spot, as they will usually include clue comparison phrasing like ‘more than,’ ‘less than,’ ‘like’ or ‘unlike,’ or ‘as/as.’ Start to take note of these clue words as you complete GMAT sentence correction practice questions.

 

Donuts ARE pretty rad, though.
Donuts ARE pretty rad, though.

 

Parallelism

Parallelism means what it sounds like: you want the related parts of your sentence to be parallel, or matching. This error is especially common in lists. Here’s a simple example of a parallelism error:

I loved to sing, to dance, and acting.

The first two items in the list are infinitive verbs (verbs preceded by ‘to’)—to sing and to dance—followed by a noun (‘acting’). The correct version of the sentence would read, “I loved singing, dancing, and acting,” or “I loved to sing, to dance, and to act.”

 

Parallelism questions ask you to make sure the appropriate parts of a sentence are parallel, or 'match.'
Parallelism questions ask you to make sure the appropriate parts of a sentence are parallel, or ‘match.’

 

Dangling/Misplaced Modifiers

Modifiers are words or phrases (in the case of the GMAT, we’re referring specifically to phrases) that modify, or describe/alter/specify, another part of a sentence. Here’s a correctly placed modifier:

Looking around fearfully, she darted into the building before anyone could see her.

Here, the modifier is ‘looking around fearfully,’ which describes ‘she.’

On the GMAT, you will encounter modifiers that aren’t in the right place. These are called dangling or misplaced modifiers. In general, remember that you want your modifier to be as close to the noun it’s modifying as possible.

A dangling modifier is one that ‘hangs’ off a sentence (usually the beginning) and doesn’t modify the right word or words. For example:

A majestic fairy tale creature, Frankenstein admired the Pegasus for her intellect.

There are many ways to describe Frankenstein, but ‘majestic fairy tale creature’ is probably not one of them. That modifying phrase should be next to the word it’s modifying (Pegasus). The new sentence would read:

Frankenstein admired the Pegasus, a majestic fairy tale creature, for her intellect.

A misplaced modifier can be a little more difficult to spot. Here’s an example:

She made her life plan to write a bestselling novel on the day she graduated from college.

This sentence might be true, but it’s rather illogical and more than a bit unrealistic. Here, the modifying phrase is ‘on the day she graduated from college,’ which is when she likely did the planning, not the day she did the writing. Right now, it’s misleading. We’d rewrite it as such:

On the day she graduated from college, she made her life plan to write a bestselling novel.

 

I've heard from Frankenstein that she's very smart.
I’ve heard from Frankenstein that she’s very smart.

 

Pronoun Usage

Pronouns stand in for nouns, and the nouns they stand in for are called antecedents. For example, in the sentence “Carol left her coat outside,” Carol is the antecedent of the pronoun her.

The most common pronoun error type on the GMAT is the ambiguous pronoun. An ambiguous pronoun is one whose antecedent is unclear:

Sarah and Julie took her dog for a walk.

In this sentence, whose dog is it? What is the antecedent of ‘her?’ Sarah or Julie? There’s no way to tell for sure what the antecedent of ‘her’ is. The correct version of the sentence would clarify this for the reader (for example, “Sarah and Julie took Julie’s dog for a walk”).

On the GMAT, look out particularly for pronouns like ‘it’ and ‘they.’ While any pronoun can be ambiguous, these are the two most likely culprits. For example:

When George dropped his vase onto the glass table, it cracked.

What cracked? The vase or the table? We can’t be sure, because we don’t know what the antecedent of ‘it’ is. We need to clarify that for the reader.

Pronoun agreement errors are also common on the GMAT. A pronoun agreement error is when a pronoun’s antecedent is plural, while the pronoun is singular, or vice versa. For example:

The male peacock spreads their feathers to impress the female peacock.

The singular antecedent ‘the male peacock’ does not agree with the pronoun ‘their.’ Instead, the sentence should be written as such:

The male peacock spreads its feathers to impress the female peacock.

This error type comes up particularly often in sentences dealing with animals. When speaking of an animal in general (‘the brown bear,’ ‘the flamingo,’), the correct pronoun is ‘it.’

 

Don't leave your pronouns a mystery. Clarify them!
Don’t leave your pronouns a mystery. Clarify them!

 

Clarity and Concision

GMAT sentence correction questions will require you not only to make sure that a sentence is grammatically correct, but also to ensure that it’s not redundant, ambiguous, awkwardly phrased, or clunky.

For example, take a look at this sentence:

At this point in time, she has knowledge and comprehension of her error.

What could be more concise in this sentence? Let’s rewrite it.

Now, she understands her error.

Notice that the two sentences mean the same thing, but the former uses unnecessary phrases like “at this point in time” when simply “now” is sufficient. When you think you have found a clarity and concision error, take note: Does the shorter sentence shift the sentence’s meaning? Or does it simply say the same thing in a cleaner and more concise way?

Remember that this doesn’t mean you should simply choose the shortest answer choice or that the shortest answer option is always the correct one. You should always check for other error types first, and eliminate answer choices that way. If you can whittle the answer options down to two, however, it’s a safe bet to choose the shorter one.

 

When it comes to the GMAT, shorter sentences are often preferable.
When it comes to the GMAT, shorter sentences are often preferable.

 

GMAT Sentence Correction Practice Questions: Examples

Let’s go through four examples of GMAT sentence correction question, covering the most common error types.

 

Practice Question 1: Subject-Verb Agreement

While larger banks can afford to maintain their own data-processing operations, many smaller regional and community banks are finding that the cost associated with upgrading data-processing equipment and with the development and maintenance of new products and technical staff are prohibitive.

(A) cost associated with

(B) costs associated with

(C) costs arising from

(D) cost of

(E) costs of

Source: http://www.mba.com/us/the-gmat-exam/gmat-exam-format-timing/verbal/sample-sentence-correction-question.aspx

This is a subject-verb agreement question; one clue is that the various answer choices include both plural and singular subject options (‘costs’ and ‘cost’). The first thing to do is to locate the subject and verb: ‘costs’ or ‘cost’ is the subject, while ‘are’ is the verb. Since we don’t have the option to change the plural verb ‘are,’ the subject must be ‘costs,’ so we can narrow down the answer choices right away to B, C, or E. Choices A and D contain subject-verb agreement errors.

Answer choices C and E contain idiomatically incorrect expressions (‘arising from’ and ‘costs of upgrading’).

Answer choice B (‘costs associated with’) is correct in terms of both subject-verb agreement and idiomatic expression.

 

Matching a subject to its corresponding verb is the first step to any subject-verb agreement question.
Matching a subject to its corresponding verb is the first step to any subject-verb agreement question.

 

Practice Question 2: Dangling Modifier

dangling modifier 2

The original sentence is an example of a dangling modifier: In answer choice A, ‘architects and stonemasons’ is incorrectly modifying ‘huge palace and temple clusters.’ So some of the other answer choices are likely to contain dangling modifiers, too. When you see a phrase followed by a comma introducing a sentence, it’s a good idea to check for a dangling modifier before doing anything else.

Answer choice B is also a dangling modifier (‘architects and stonemasons’ is incorrectly modifying ‘without of benefits of animal transport’).

Answer choice D is also a dangling modifier and contains clarity and concision errors, beginning with the passive construction ‘there were built.’

Answer choice E isn’t a dangling modifier, but it’s still incorrect: It contains the grammatically incorrect ‘were the Maya who.’

Answer choice C corrects the modification error: ‘Architects and stonemasons’ is correctly modifying ‘the Maya.’ It is also preferable because it contains the active verb ‘built’ rather than the passive ‘were built,’ which is in several of the other answer choices. Answer choice C (‘the Maya…’) is correct.

 

pencil-918449_1280
In dangling modifier questions, you’re looking to match the correct noun with its appropriate modifying phrase.

 

Practice Question 3: Illogical Comparison, Idioms, Clarity and Concision

Many sentence completion questions contain more than one error. Several error types apply to this question.

dangling modifier

The original sentence, answer choice A, is awkwardly phrased and wordy (a clarity and concision issue) and contains an illogical comparison (‘computer skills’ to ‘disinclination’). Look for illogical comparison issues when you see clue words like ‘as,’ ‘more,’ ‘less,’ ‘than,’ or ‘like.’

Answer choice B illogically compares ‘computer skills or other technical skills’ to ‘many people.’

Answer choice C has clarity and concision errors; the phrase ‘analytical skills bring out a disinclination’ is idiomatically incorrect. Answer choice E contains the wordy construction ‘Many people have a disinclination.’

Answer choice D clearly compares ‘computer skills or other technical skills’ to ‘analytical skills.’ It is also clearer and more concise than the other choices. The correct answer is D (“Many people, willing…”).

 

When it comes to the GMAT, don't compare apples and oranges!
When it comes to the GMAT, don’t compare apples and oranges!

 

Practice Question 4: Pronoun Usage, Clarity and Concision

This question contains both pronoun usage and clarity/concision problems.

sentence correction

Answer choices A and B are both unclear and too wordy, but answer choice B also contains a pronoun agreement error. ‘Crab,’ a singular noun, is not the correct antecedent for the plural ‘their.’ It’s a good idea to look for pronoun usage problems when you see potentially problematic pronouns like ‘it’ or ‘their’ in the original sentence or an answer choice, or when you see several different options for pronouns in the answer choices.

Answer choice E also contains a pronoun agreement error: ‘Crab’ is a singular antecedent, while ‘they’ is a plural pronoun.

Answer choice D contains an idiomatic error (‘because of living’ is not a correct idiomatic expression).

Answer choice C corrects the pronoun error and appropriately matches ‘crab’ with the singular pronoun ‘it.’ The correct answer is C (‘because it lives’).

 

Clarity is the name of the game for sentence correction questions.
Clarity is the name of the game for sentence correction questions.

 

Sentence Correction GMAT Question Tips: Key Strategies

Remember that the first choice is “no change.” This is a simple tip, but it will save you a bit of reading time. It’s not labeled as no change, but the first answer choice is always the same as the given sentence. Try to identify the potential error type as soon as you read it (an introductory phrase followed by a comma, for example, often signals a dangling modifier). If you can’t, quickly skim the other answer choices for clues.

Shorter is often better. Particularly if you’re down to two answer choices in your process of elimination, choose the shorter option! Remember, the GMAT loves concision. If you’re fairly certain that two options are technically (grammatically) correct, the shorter answer choice is a safe bet. The shortest answer won’t always be the right one, but it’s a good guideline for clarity and concision questions in particular. Does one answer choice say the same thing as another, but uses extraneous words? There’s a good chance it’s not the best choice.

Cross out extraneous words and phrases. This is something you can use your scratch pad (or, at home, your sentence correction worksheet) for. Particularly for subject-verb agreement questions, try rewriting the sentence and crossing out all the extra words between the subject and verb. Prepositional phrases between the subject and verb, for example, are often used to confuse you. Isolating the key words will cut down on confusion and potential timing issues. It will also help you eliminate incorrect answer choices more confidently.

Take about a minute per question. Since you have 65 minutes to complete the verbal section and 36 questions to complete, and reading comprehension and critical reasoning questions tend to take a bit longer (one and a half to two minutes each), you should plan to spend around one minute on each sentence correction question. You should keep a sentence correction worksheet to log the amount of time you spend on each question.

 

Practice crossing out extraneous words in sentences on the GMAT.
Practice crossing out extraneous words in sentences on the GMAT.

 

GMAT Sentence Correction Tips: How to Prepare

Brush up on basic grammar terms, rules, and concepts. You won’t be tested directly on grammar terms (such as parts of speech) on the GMAT, but it can be very helpful to review them before the exam (especially if you’re rusty) as part of your GMAT sentence correction practice. If you’re thinking in terms of subjects and verbs when you’re faced with a subject-verb agreement question or can quickly tell the difference between the past perfect and the present perfect for a verb tense question, it becomes a lot easier to quickly correct any technical errors in a given sentence. Remember that you need to focus specifically on GMAT sentence correction rules, not just on grammar rules in general.

Learn to recognize error types. Recognizing sentence correction error types quickly and accurately will improve both your timing and your performance. GMAT sentence correction practice should always include labeling error types as you go along.

Once you recognize an error type, you’ll know much more quickly what to look for and how to solve the problem. For example, a subject-verb agreement question means you’re looking to isolate the subject and verb and eliminate the answer choices that match plural subjects with singular verbs, or vice versa, right off the bat. A dangling modifier question will require you to look for the noun immediately following a modifying phrase to make sure it’s being modified accurately. Knowing how to tackle each kind of error type will help you streamline your approach to each question. Use the list above and label each sentence correction question as you review. With practice, you’ll learn to recognize them right away and apply GMAT sentence correction rules to your approach to each question.

Practice with real sentence correction GMAT questions regularly. During GMAT sentence correction practice sessions, use the GMATPrep software or the resources in GMAT Verbal Practice to practice specifically with sentence correction questions and review them. This will help you get comfortable with the format of each error type and with the various GMAT sentence correction rules. As soon as possible, as you complete GMAT sentence correction practice questions, hone in on your weakest areas and focus your prep time on them. Keep track of your most common errors on a sentence correction worksheet.

Learn the most common idioms. Especially if you’re not a native English speaker, consider learning the most common idioms that crop up on the GMAT and incorporating that work into your GMAT sentence correction practice. Keep a running list of idioms that aren’t familiar to you as you practice, and make sure you know those as well.

 

whats-next-1462747_1280

What’s Next?

If you’d like similar breakdowns of other GMAT question types, check out our top tips for data sufficiency and critical reasoning questions.

For a comprehensive overview of the GMAT and how to prepare for each section, including more GMAT sentence correction tips and GMAT sentence correction practice questions, go to our complete GMAT study guide.

Nervous about the rest of the GMAT verbal section? Our complete guide to GMAT verbal will help you ace the reading comprehension and critical reasoning portions of the exam, in addition to offering even more GMAT sentence correction tips.

The post 8 Top Tips for GMAT Sentence Correction Questions appeared first on Online GMAT Prep Blog by PrepScholar.

]]>
Subjunctive Mood GMAT Questions: 2 Rules You Must Know /gmat/blog/subjunctive-mood-gmat/ Fri, 14 Apr 2017 00:00:15 +0000 http://gmat.psblogs.com/?p=2381 Never even heard of the subjunctive mood before? Don’t worry, you’re not alone: this mood is sometimes taught to students studying a romance language, but it’s rarely spelled out in English class. But even though you may not know it, you’ve likely been using—and sometimes misusing—the subjunctive mood in your own speaking and writing. Unfortunately, GMAT sentence … Continue reading "Subjunctive Mood GMAT Questions: 2 Rules You Must Know"

The post Subjunctive Mood GMAT Questions: 2 Rules You Must Know appeared first on Online GMAT Prep Blog by PrepScholar.

]]>
Feature_girlmakingwishflower

Never even heard of the subjunctive mood before? Don’t worry, you’re not alone: this mood is sometimes taught to students studying a romance language, but it’s rarely spelled out in English class. But even though you may not know it, you’ve likely been using—and sometimes misusing—the subjunctive mood in your own speaking and writing.

Unfortunately, GMAT sentence correction questions are going to test your ability to explicitly recognize (and correct) the subjunctive mood. In general, the GMAT loves to test you on sentences that sound right but aren’t, and the subjunctive mood is perfect for this agenda. So, in this post, we’ll go over everything that you need to know about the subjunctive mood for the GMAT, including all the kinds of subjunctive mood GMAT questions you’ll encounter and an in-depth analysis of how to recognize and approach them. By the end, you’ll be able to crush any subjunctive mood GMAT question that comes your way.

 

This bear is in a bad mood—but that's not the kind of mood we're talking about!
This bear is in a bad mood—but that’s not the kind of mood we’re talking about!

 

First of All, What Is a “Mood” (Versus a Tense)?

Great question! You’re probably more familiar with verb tenses than you are with moods. The basic distinction is that tense refers to time, while mood refers to the speaker’s attitude or the manner of expression.

Essentially, a verb’s tense specifies past, present, or future. There are more subdivisions within this (like the perfect past, the pluperfect, etc.), but that’s the basic framework. Conversely, a verb’s mood refers to how the thought gets expressed. The three main moods are the indicative mood, the imperative mood, and the subjunctive mood.

The indicative mood is the most common, and it’s what we’d consider to be the normal or standard mode of expression. It’s used for assertions and questions—which covers a lot of ground. (“What time is it?” “She’s headed out to buy groceries.”)

The imperative mood is less common, but it’s also fairly easy to comprehend. It’s used to express a command or an order. (“Stop right there! Do what I tell you! Listen to me!”) Note that these are complete sentences, even though there doesn’t appear to be a subject: the “you” is implied in the command.

Here’s a nice example: “I like to write poems” is in the indicative mood, but “Write me a poem, Jess” is in the imperative mood.

The subjunctive mood is used to express conditional or imaginary situations, meaning scenarios that are doubtful, hypothetical, or otherwise run contrary to fact. (“If I were you, I would share my toys with my little sister.”)

The subjunctive is also used after the word “that” following verbs that express suggestions, demands, or recommendations. (“I suggest that he leave for the hike early tomorrow morning to catch the sunrise.”)

In all use cases, the subjunctive mood indicates that the situation occurs “outside” of time: it’s not a real scenario. To illustrate what this looks like in action, we’ll delve into the kinds of sentences that use the subjunctive mood below.

Don't worry if the subjunctive mood is still confusing—we're just getting started!
Don’t worry if the subjunctive mood is still confusing—we’re just getting started!

 

What Is the Subjunctive Mood?

As stated above, in English, the subjunctive mood is used to explore non-real (conditional or imaginary) situations. Within this, there are two main categories for the subjunctive mood: commands/suggestions (with a command verb + “that”), and hypotheticals/wishes (with “were”).

It’s helpful to divide the subjunctive this way because each category has its own key rule that you’ll need to know for the GMAT. So make sure you are familiar with the below use cases before moving on to the next section.

 

Category 1: Suggestions, Demands, and Necessity (With “That”)

The subjunctive mood is used to express suggestions and recommendations with the word “that”:

“I suggest that you get here early to park your car.”“I recommend that they take the bus home.”

“The job requires that candidates be experienced in Javascript.”

Similarly, it is used to express demands that aren’t in the imperative mood (meaning that the demand isn’t expressed directly at the listener):

“We insist that you leave at once.” (as opposed to “Leave at once!”, which is in the imperative mood.)“I demand that he be here in time for my performance.”

“We demand that the traitor be executed.”

“They are insisting that he move to Denver to manage the new office.”

It is also used to express necessity in the impersonal (“it is necessary that ” versus “I need you to”):

“It is essential that you come to class today.”“It’s necessary that the tour group wake up before sunrise.”

“It’s crucial that he find a subletter to take over the lease for him.”

 

Category 2: Hypotheticals and Wishes (With “Were”)

The subjunctive is also used to express hypotheticals, situations that are contrary to fact, and situations that are unlikely to come true. This usage employs “were,” and then sometimes “would”:

“If she were to beg for forgiveness, would you show her mercy?”“Suppose that I were to fly into town for the concert.”

“The doctor dismissed my cough as though it were harmless.”

“If I were the Queen of England, I would bake everyone muffins.”

“If I were you, I wouldn’t throw my birthday party on a Monday night.”

Similarly, it is used to express wishes and desires:

“I wish I were able to attend your birthday party.”“I wish I were done studying for the GMAT.”

“She wishes that the sun were out.”

“If I were a rich man (daidle deedle daidle daidle daidle deedle daidle dum…)”

What do all of these many uses for the subjunctive mood have in common? They all express situations that are not real or factual, and thus exist outside of time and tense: they’re either conditional on something else happening, or they’re hypothetical, or they’re something that the speaker wants to happen, or they’re contrary to fact and totally impossible, or they should happen in the future but aren’t set in stone.

 

If I were a unicorn, I would not exist in real life.
If I were a unicorn, I would not exist in real life.

 

The 2 Key Subjunctive Mood GMAT Rules

For GMAT sentence correction questions, whenever you see a sentence that fits into one of the above categories, you know that you’re dealing with the subjunctive. There are really only two rules that you need to know for these questions.

 

Rule 1: Demand/Suggestion Subjunctives Always Take the Base Form of the Verb After “That”

The base form of a verb is the same as the infinitive form, but without the “to.” It’s not conjugated—as in, it’s not in first, second, or third person singular or plural—and it has no tense. For example, “is,” “was,” “were,” “are,” and “am” are all different conjugations and tenses of the verb “to be.” So the base form of that verb is simply “be” (the infinitive “to be” without the “to”).

Always use the base form of the verb with subjunctive mood demands and suggestions:

“I require that the top candidate send me her cover letter.””The mayor demanded that the residents be evacuated immediately.”

“The doctor suggests that Richard avoid heavy lifting for six to eight weeks.”

This rule is why it’s easier to “see” a demand subjunctive in action when it’s not in the second person: many second-person conjugations are the same as the base form of the verb:

“I suggest that you wake up before dawn.”

This doesn’t look weird to us, and it correctly utilizes the subjunctive mood. But look at the same sentence with a third-person subjunctive clause:

“I suggest that he wake up before dawn.”

This does sound a little strange, because we’re used to saying “he wakes up”—which would not be correct in this example, as it’s a subjunctive mood suggestion and thus takes the base form of the verb “wake.”

Here’s another example:

“I insist that Harry is there.”

This might sound right, but it’s written in the indicative mood, which means that it’s only grammatically correct if I am trying to convince someone that Harry is currently “there.” (Maybe I’m pointing to Harry’s ghost and am insisting to my listeners that Harry is actually present, even though they can’t see him?!)

Now look at the sentence in the subjunctive:

“I insist that Harry be there.”

This is correct if I’m demanding that Harry show up to “there” (wherever “there” is). The sentence is written in the subjunctive mood and it’s probably what I actually mean when I say it (I hope…spooky!).

This rule holds up even with passive constructions of verbs:

“The judge ordered that the witness be imprisoned for perjury.”

 

I insist that Harry IS there—right there in that creepy old living room...
I insist that Harry IS there—right there in that creepy old living room…

 

Rule 2: Hypothetical/Wish Subjunctives Use “Were”

Some of the sample hypothetical/wish sentences above may have seemed weirdly constructed to you. You might have been wondering why I wrote, “I wish I were done studying,” and not “I wish I was done studying.”

I did so because, the past subjunctive “were” is used for sentences with hypotheticals/wishes. 

In other words, use “were” when the situation is not reality or not yet reality—even when you’re not speaking in the past tense:

“I wish I were done studying.”

This conveys that I want to be done studying, but I am not yet done studying.

The same goes for the hypothetical scenarios:

“If I were the Queen of England, I would bake muffins for everyone.”

This is a hypothetical that runs contrary to fact and isn’t even possible in the future (unless Prince Harry wants to leave me a comment below). It falls in the same broader category of subjunctive mood uses—hypotheticals and wishes—so it too takes “were.”

“Was” is only correct when the situation does exist in reality:

“I wanted to know if Anju was looking for a new roommate.”

In this case, Anju either is or isn’t looking for a roommate. It’s a real possibility, not a desire that isn’t possibly real like in the examples above, so it’s not in the subjunctive mood.

 

What Kinds of GMAT Subjunctive Mood Questions Are There?

Subjunctive mood GMAT questions test you on both categories of subjunctive mood use cases: suggestions/demands and hypotheticals/wishes. You’ll be expected to recognize when a sentence should be using the subjunctive, and to be able to correct it accordingly using the two rules above.

We’ll go over some example subjunctive mood GMAT questions below, so you can see what this looks like in action.

 

If I were you, I'd adopt all of these cute kittens together!
If I were you, I’d adopt all of these cute kittens together!

 

Example GMAT Subjunctive Mood Questions

Below, we analyze an example of each kind of subjunctive mood question. We also give you one example of a tricky sentence that does not need the subjunctive mood, so that you can learn to recognize subjunctive mood “fakeouts” as well.

 

Example Subjunctive Mood GMAT Question #1: Suggestions/Demands

The following is an official GMAT question:

The budget for education reflects the administration’s demand that the money is controlled by local school districts, but it can only be spent on teachers, not on books, computers, or other materials or activities.

(A) the money is controlled by local school districts, but it can only be spent

(B) the money be controlled by local school districts, but it allows them to spend the money only

(C) the money is to be controlled by local school districts, but allowing it only to be spent

(D) local school districts are in control of the money, but it allows them to spend the money only

(E) local school districts are to be in control of the money, but it can only spend it

The phrase “demand that” indicates that this sentence should be in the subjunctive mood, so it will take the base form of the verb. In this case, the verb is “to be controlled” (“control” in the passive construction), so it’s going to take the base form of “to be” with controlled. So, we’re looking for the option that uses “be controlled.”

A, D, and E can be crossed off immediately, as they are written in the indicative and thus don’t use the base form of “be.” (C) looks good at first, but “is to be controlled” is still actually indicative (“is”). Get rid of it as well.

Read with (B), the sentence reads: “The budget for education reflects the administration’s demand that the money be controlled by local school districts, but it allows them to spend the money only on teachers, not on books, computers, or other materials or activities.” This is a correct use of the subjunctive and is the answer.

 

A subjunctive mood sentence about a school budget? How meta!
A subjunctive mood sentence about a school budget? How meta!

 

Example Subjunctive Mood GMAT Question #2: Hypotheticals/Wishes

The following is a PrepScholar GMAT question very much like one you might see on a real test:

As if the doubly reinforced window was not enough, the jail also uses panoptical surveillance and a barbed wire fence around the perimeter to ensure that the dangerous prisoner can never escape.

(A) the doubly reinforced window was not enough, the jail also uses panoptical surveillance

(B) the window having been doubly reinforced was not enough, the jail also uses panoptical surveillance

(C) the doubly reinforced window is not enough, the jail also uses surveillance of the panoptical variety,

(D) the doubly reinforced window were not enough, the jail also uses panoptical surveillance

(E) the doubly reinforced window is not to be enough, the jail also uses panoptical surveillance

At first glance, this looks like an issue of tense: should it be “was not enough” or “is not enough”? But that’s actually not the problem with the sentence as written. Rather, this sentence poses a hypothetical: we’re exploring the unlikely possibility that the doubly reinforced window isn’t strong enough to ensure that the prisoner can’t escape. As such, this scenario doesn’t exist in reality and it needs to be put in the subjunctive mood.

Only (D) correctly employs the past subjunctive “were.” (D) is the answer.

 

Extra Example: A GMAT Question that DOES NOT Need the Subjunctive Mood (But Might Trick You)

The following is an official GMAT question:

A wildlife expert predicts that the reintroduction of the caribou into northern Minnesota would fail if the density of the timber wolf population in that region is more numerous than one wolf for every 39 square miles.

(A) would fail if the density of the timber wolf population in that region is more numerous

(B) would fail provided the density of the timber wolf population in that region is more

(C) should fail if the timber wolf density in that region was greater

(D) will fail if the density of the timber wolf population in that region is greater

(E) will fail if the timber wolf density in that region were more numerous

You might be tempted to pick (E), as it looks like it “corrects” the sentence by using the past subjunctive “were” instead of the present indicative “is.” However, the subjunctive is only used for non-real situations: things that haven’t happened in the past, are not happening now, and aren’t likely to happen in the future.

In this case, we have a “wildlife expert” making a prediction about what is likely to happen in the future—based on what is real about right now. In other words, the one-wolf-per-39-miles population size is a current possible reality, unlike my becoming the Queen of England. Therefore, “to be” should not take hypothetical subjunctive mood and is correct in the indicative mood, present tense.

However, the sentence isn’t entirely correct as written: the prediction for what will happen to the caribou if this scenario turns out to be true should be described with the future tense (“will fail if”), not the conditional tense (“would fail if”). It also needs to use “greater than” as opposed to “more numerous than,” since we’re talking about the collective singular noun “population.”

With (D), the sentence reads: “A wildlife expert predicts that the reintroduction of the caribou into northern Minnesota will fail if the density of the timber wolf population in that region is greater than one wolf for every 39 square miles. This correctly puts the verb “fail” in the indicative mood, future tense; keeps “is” in the indicative mood, present tense; and changes “more numerous than” to “greater than.” Hence, (D) is the correct answer.

 

This wolf could be from Northern Minnesota.
This wolf could be from Northern Minnesota.

 

3 Key Tips for Subjunctive Mood GMAT Questions

Here’s how to nail every kind of subjunctive mood question that you’ll see on the GMAT.

 

Tip #1: Memorize The Two Subjunctive Mood Rules

This one is a given: there are only two main subjunctive mood rules that you need to know for the GMAT: to use the base form of the word with “that” subjunctives (demands and suggestions), and to use the past subjunctive “were” for sentences with hypotheticals/wishes. Commit them to memory now.

 

Tip #2: Write Down Subjunctive Mood Examples That You Read

Look out for the subjunctive mood in sources like The Atlantic and The New York TimesWhen you see it in action, write down the example you came across in a notebook or in the notes section of your iPhone. This will help you become more familiar with its many uses.

You should also start implementing the subjunctive rules in your everyday writing. You’ll be surprised at how many instances of the subjunctive mood you’ve been missing—and how it easy it is to get it right with just those two rules!

 

Have you caught any subjunctive mood uses in your daily reading?
Have you caught any subjunctive mood uses in your daily reading?

 

Tip #3: Practice, Practice, Practice!

Subjunctive mood questions aren’t very common, but they will come up in your GMAT sentence correction practice from time to time. The more you practice with GMAT sentence correction questions, the more you’ll become familiar with complex sentences in the subjunctive mood, and the better you’ll get at spotting them on the real test. You’ll also get better at spotting the kinds of sentences that shouldn’t be in the subjunctive mood (but seem like they should be at first), so you won’t be fooled by questions like the last example above.

One great resource for practicing sentence correction questions (including the subjunctive mood ones) is The Official Guide for GMAT Verbal Review. The book includes 300 official practice questions from retired GMATs, access to an accompanying site where you can customize sets of practice questions, reviews of grammar and reading comprehension fundamentals, and online videos with tips and strategies specific to the verbal section.

Another solid option is the Kaplan GMAT Verbal Workbook, which contains about 220 unofficial GMAT verbal questions, with nearly 100 of those questions devoted to sentence correction.

 

What’s Next?

The subjunctive mood isn’t the only quirky grammatical concept tested by the GMAT, so be sure to review our guide to the six grammar rules that you need to know for the GMAT.

If the Verbal section in general is challenging for you, head to our post on how to master the three Verbal question types.

We can also help you find the best GMAT Verbal practice once you’re ready to dive in.

Happy studying!

 

The post Subjunctive Mood GMAT Questions: 2 Rules You Must Know appeared first on Online GMAT Prep Blog by PrepScholar.

]]>
What Is on the GMAT? Expert Guide to GMAT Subjects /gmat/blog/what-is-on-the-gmat/ Sat, 08 Apr 2017 21:00:21 +0000 http://gmat.psblogs.com/?p=1876 If you’re applying to business school, you’re probably going to have to take the GMAT, or the Graduate Management Admissions Test. Preparing for the GMAT requires a lot of time and effort, but you can’t create a successful study plan without knowing exactly what topics it tests you on. So if you’re just beginning your … Continue reading "What Is on the GMAT? Expert Guide to GMAT Subjects"

The post What Is on the GMAT? Expert Guide to GMAT Subjects appeared first on Online GMAT Prep Blog by PrepScholar.

]]>
feature_chalkboardmath

If you’re applying to business school, you’re probably going to have to take the GMAT, or the Graduate Management Admissions Test. Preparing for the GMAT requires a lot of time and effort, but you can’t create a successful study plan without knowing exactly what topics it tests you on.

So if you’re just beginning your GMAT prep, you’re probably wondering: what are the major GMAT subjects? What skills does the GMAT test, and what’s the basic format of the exam? What kinds of questions will I see? What topics should I be studying, and do I have to memorize anything? In short: what is on the GMAT?

Luckily, you’ve come to the right place! In this guide, we’ll break down exactly what you need to know for each section of the test, explain the different types of questions and skills they focus on, and then wrap up with some general tips on how to prep.

 

GMAT Subjects: What You Need to Know Overall

Each section is different, but what does the GMAT test you on as a whole? In a sentence, the GMAT tests your academic preparedness and readiness to succeed in MBA and other graduate business degree programs. It’s the most widely used test by both business schools and their applicants (read our guide to the GMAT versus the GRE for more), as it has proven to be a highly accurate metric for predicting how well someone will do in an MBA program.

Note that the GMAT tests your preparedness for business school: not your intelligence, and not whether you already have business skills and knowledge.

More specifically, the GMAT tests your analytical, writing, quantitative, verbal, and reading skills—in other words, what you’ve learned how to do throughout your academic career thus far.

 

You don't need any prior knowledge of business concepts for the GMAT.
You don’t need any prior knowledge of business concepts for the GMAT.

 

Exam Format Basics: The 4 Sections of the GMAT

Now that you’ve gotten the broad strokes of GMAT subjects, let’s quickly go over the basic structure and the four different sections that you’ll see on the test. Then we’ll get into the exact subjects, question types, and skills for each section.

First, you should know that all four sections of the GMAT are timed, and all are taken consecutively on a computer at one of many testing locations around the world. In every section, you can only see one question at a time, and you cannot go back to re-answer or view previous questions: once you’ve answered and clicked “submit” for that question, it’s final.

The sections of the test are always in the same order, as laid out below:

The first section is the Analytical Writing Assignment. Here, you’ll get a short prompt that makes some kind of argument, and you’ll have 30 minutes to write a brief essay analyzing the given argument.

The second section is the Integrated Reasoning section. Here, you’ll get 30 minutes to complete 12 multi-part questions.

The third section is the Quantitative section. This is the big math section of the GMAT, and you’ll have 62 minutes to complete 31 multiple-choice questions, each with five different answer options. The Quantitative section is computer adaptive, meaning that the computer adjusts to your skill level as you go: starting from the very first question onward, subsequent questions are adjusted to an easier level if you answer incorrectly, or they are adjusted to a more difficult level if you answer correctly.

After Quant, you’ll hit the fourth and final section of the GMAT: the Verbal section. This is the big reading and grammar section of the test, and you’ll have 65 minutes to complete 36 multiple-choice questions, each with five different answer options. Like the Quant section, it is computer adaptive.

This information is distilled in the table below:

GMAT Test Section # of Questions General Skill(s) Timing
Analytical Writing Assessment 1 prompt featuring an argument Analysis of given argument 30 minutes
Integrated Reasoning 12 Integrating information from different sources 30 minutes
Quantitative 31 Math 62 minutes
Verbal 36 Reading and writing 65 minutes
Total Exam Time 3 hours, 7 minutes 

Now that we know the basic structure and format of the GMAT, let’s dive right into what skills, subjects, topics, and question types are in each section.

 

The Quant and Verbal sections of the GMAT are computer adaptive—meaning they adjust to your ability level as you go.
The Quant and Verbal sections of the GMAT are computer adaptive—meaning they adjust to your ability level as you go.

 

What Is on the GMAT Analytical Writing Assessment?

The AWA is the “essay” portion of the GMAT; however, it likely differs greatly from most of the academic essays you’ve written thus far.

 

GMAT Topics and Skills Covered

According to the official GMAT website, the Analytical Writing Assessment (or AWA) measures “your ability to think critically and to communicate your ideas.” You will be given a single one-paragraph prompt containing some kind of argument. The prompts often center on debates from the business or political worlds and are sourced from the editorial and op-ed sections of magazines and newspapers, annual company reports, memorandums, proposals and beyond.

 

Question Types

You have only one task here: to write a critique of the given argument. The critique should contain an in-depth analysis of the underlying reasoning of the argument—including any flaws in reasoning that the author makes. Do not present your own views on the argument; stick to deconstructing the prompt.

The directions are always the same for the AWA. I’ve pasted them for you below:

Discuss how well reasoned you find this argument. In your discussion be sure to analyze the line of reasoning and the use of evidence in the argument. For example, you may need to consider what questionable assumptions underlie the thinking and what alternative explanations or counterexamples might weaken the conclusion. You can also discuss what sort of evidence would strengthen or refute the argument, what changes in the argument would make it more logically sound, and what, if anything, would help you better evaluate its conclusion.

Though a formal understanding of logic is not required, it’s helpful to be well-versed in persuasive writing and to understand concepts like counterargument and rebuttal, faulty reasoning, and claims and evidence.

 

For the AWA, it helps to have some knowledge of how arguments are constructed
For the AWA, it helps to have some knowledge of how arguments are constructed.

 

What Is on the GMAT Integrated Reasoning Section?

The challenging Integrated Reasoning section tests multiple skills at once—hence the name Integrated Reasoning!

 

GMAT Topics and Skills Covered

According to the official GMAT website, the Integrated Reasoning (or IR) section measures “your ability to evaluate information presented in multiple formats from multiple sources—skills you need to succeed in our technologically advanced, data-driven world.” It’s a relatively new section that was added to the GMAT in 2012 as a way of better assessing the kinds of complex reasoning skills that you’ll need both in business school and in your career afterward.

Here is the official list of the four skills tested:

  • Synthesizing information presented in graphics, text, and numbers
  • Evaluating relevant information from different sources
  • Organizing information to see relationships and to solve multiple, interrelated problems
  • Combining and manipulating information from multiple sources to solve complex problems

Essentially, you will have to integrate data from different sources to solve complex, multi-step problems. The vast majority of the actual concepts tested in the Integrated Reasoning section are covered in the Quant and Verbal sections, which we get into below. The only additional concepts you’ll be tested on in IR are:

  • The interpretation of different kinds of charts and graphs, and
  • The basic functionality of column-sorting in spreadsheets.

In IR, unlike any other section of the GMAT, you have access to a very basic, click-operated calculator that is available on your screen. You probably won’t need to use it for every question, but it’s there for you when you do need it.

 

A fundamental understanding of charts and graphs is necessary for the Integrated Reasoning section!
A basic understanding of charts and graphs is key for the Integrated Reasoning section!

 

Question Types

The Integrated Reasoning section is a new and unique part of the GMAT; as such there are four very specific types of questions that you’ll encounter.

 

#1: Graphics Interpretation

In these questions, you’re given a graph or chart and some statements that pertain to it, and you have to select the option from one or more drop-down menus to make each statement accurate. It’s similar to a “fill in the blank” style question, but with drop-down menus limiting the number of options.

body_GraphicsInterpretationScreenshot

 

#2: Two-Part Analysis

Here, you’re given a paragraph of information and a two-column table that represents a two-part solution. Each column has a question and several answer options. You have to select one answer from each column to solve the problem.

body_twopartanalysisscreenshot

 

#3: Table Analysis

Here, you’re given a table of data that you can sort by data type, like in an Excel or Google Sheets spreadsheet. You have to organize the data to figure out whether certain conditions are met. Each question will have several statements with opposing answers (e.g., yes/no, true/false, inferable/not inferable) and you have to select one answer for each statement in the question.

body_tableanalysis

 

#4: Multi-Source Reasoning

These questions have a split-screen format. On the right side of the screen, you’re given multiple “tabs” that you can navigate among. Each tab contains different data—most often a few paragraphs of verbal information (with perhaps a chart). On the left side of the screen, multiple-choice questions are presented one by one, and you have to figure out what information from which tabs you need to answer each question.

body_MultiSourceReasoningScreenshot

An important note: even though most Integrated Reasoning questions have more than one part to them, there is no partial credit given. You have to answer all parts of a single question correctly to receive credit. If, for example, you correctly answered one “true/false” statement on a table analysis question but missed another statement, you would not receive any credit for the question.

 

What Is on the GMAT Quant Section?

The Quant section is the main math section of the GMAT, and marks the start of the multiple-choice portion of the exam.

 

GMAT Topics and Skills Covered

According to the official GMAT site, the Quantitative (or Quant) section measures “your ability to analyze data and draw conclusions using reasoning skills.” The actual math subjects tested are all from what you learned in middle and high school math: algebra, arithmetic, geometry, and word problems within these three areas. You don’t need trigonometry and you certainly don’t need calculus.

Here are all the math topics that are covered in the Quant section, organized by subject area:

Arithmetic

  • Properties of Integers
  • Fractions
  • Decimals
  • Real Numbers
  • Ratio & Proportions
  • Percents
  • Powers & Roots of Numbers
  • Descriptive Statistics
  • Sets
  • Counting Methods
  • Discrete Probability

Algebra

  • Simplifying Algebraic Expressions
  • Equations
  • Solving Linear Equations with One Unknown
  • Solving Linear Equations with Two Unknowns
  • Solving Equations by Factoring
  • Solving Quadratic Equations
  • Exponents
  • Inequalities
  • Absolute Value
  • Functions

Geometry

  • Lines
  • Intersecting Lines and Angles
  • Perpendicular Lines
  • Parallel Lines
  • Polygons (Convex)
  • Triangles
  • Quadrilaterals
  • Circles
  • Rectangular Solids and Cylinders
  • Coordinate Geometry

Word Problems

  • Rate Problems
  • Work Problems
  • Mixture Problems
  • Interest Problems
  • Discount
  • Profit
  • Sets
  • Geometry Problems
  • Measurement Problems
  • Data Interpretation

 

No calculators allowed on the Quant section, so be prepared to do mental math!
No calculators allowed on the Quant section, so be prepared to do mental math!

 

Question Types

There are two types of questions on the Quant section: problem solving questions and data sufficiency questions.

Note that there is not a fixed number of questions in either category—it varies from GMAT to GMAT. You’ll also never tackle all of one and then all of another: they are interspersed with each other randomly throughout. With that said, let’s dive into what you’ll see in each type.

 

Problem Solving

Each problem solving question gives you a problem followed by five multiple-choice answer options, and you have to work to solve the problem and pick the correct answer from among the options.

 

Data Sufficiency

Each data sufficiency questions consists of a question followed by two statements, labeled 1 and 2. Your task is to determine whether the information provided in either each statement alone or the two statements together is sufficient (or enough) to answer the question.

This is also then followed by five multiple-choice answer options; however, unlike in problem solving questions, the five answer options for data sufficiency questions are always exactly the same. We have copied them below:

(A) Statement (1) ALONE is sufficient, but statement (2) alone is not sufficient.
(B) Statement (2) ALONE is sufficient, but statement (1) alone is not sufficient.
(C) BOTH statements TOGETHER are sufficient, but NEITHER statement ALONE is sufficient.
(D) EACH statement ALONE is sufficient.
(E) Statements (1) and (2) TOGETHER are NOT sufficient.

 

What Is on the GMAT Verbal Section?

If the Quant section can be described as the main math section of the GMAT, the Verbal can be described as the main reading and writing section.

 

GMAT Topics and Skills Covered

According to the official GMAT site, the Verbal section measures “your ability to read and understand written material, to evaluate arguments, and to correct written material to conform to standard written English.” You’ll have to identify logical relationships between ideas and concepts, summarize and paraphrase passages, draw inferences, point out flaws in assumption and reasoning, and correct improper grammar or syntax.

If you like to read, the Verbal section is for you!
If you like to read, the Verbal section is for you!

 

Question Types

There are three different kinds of question types on the Verbal section: reading comprehension, critical reasoning, and sentence correction.

 

Reading Comprehension

The reading comprehension portion of the GMAT consists of four short (250-350 word) passages, each with three to four questions associated with it. They are sourced from textbooks, journals, periodicals, scholarly articles, and the like, centering on topics from the natural and social sciences, the humanities, and the business world. Note that no outside knowledge is expected in any of these domains—all of the information you will need to answer the questions is in the passages themselves.

There are several different kinds of reading comprehension questions:

  • Main idea: asks you to accurately summarize the main idea or main purpose of the passage.
  • Supporting ideas: asks you about supporting details, facts, or descriptions.
  • Inferences: asks you about what is implied but not explicitly stated in the passage.
  • Analyze logic and organization: asks you about how the passage is constructed; how the author persuades the reader; to identify underlying assumptions, strengths and weaknesses of the argument; or to point out potential counterarguments.
  • Apply info from passage to a context outside of passage: asks you to identify hypotheticals, similar examples, or if the author would agree with a given statement.
  • Style and tone: asks you to deduce the author’s attitude toward a specific idea or describe the overall tone of the passage.

Occasionally, you may encounter a question asking you to identify quant concepts as presented in writing in the passages, but the quant concepts that appear on such questions are fairly simple compared to what you see on the Quant section itself.

 

Critical Reasoning

Critical reasoning questions measure the reasoning skills you use when forming arguments, evaluating arguments, and forming or evaluating a plan of action. As with reading comprehension, you are given a passage drawn from a variety of subject areas and sources, and you are not expected to have any prior knowledge of any of them. The passages will always be making some kind of argument, and the questions you encounter will test how well you understand this argument. The three sub-categories these questions fall into are:

  • Argument construction: asks you to recognize basic structure, conclusions, assumptions, well-supported hypotheses, or parallel situations.
  • Argument evaluation: asks you to analyze reasoning and methodology, point out errors, or identify what would strengthen or weaken the argument.
  • Form/evaluate a plan of action: asks you to compare the effectiveness of different plans of action, identify what would strengthen or weaken success of a plan of action, or discern assumptions underlying a plan of action

As with the AWA, you don’t need to know the rules or terminology of formal logic to answer these kinds of questions, but it helps to be well-versed in the tactics of persuasive writing.

 

Sentence Correction

Sentence correction questions test your proficiency in the English language—namely your ability to correct a given sentence to make it clear, concise, and grammatically correct.

In these questions, you’re given a sentence, part or all of which is underlined. You have five multiple-choice answer options for re-phrasing the underlined portion, and you must pick the answer that makes for the best sentence.

The broad categories of grammar and syntax rules that the GMAT tests are:

  • Agreement (noun-verb and pronoun-antecedent)
  • Diction (standards/conventions and commonly misused words—EG, “among” versus “between”)
  • Sentence construction (independent and dependent clauses; phrases; what counts as a noun/subject and what counts as a verb/predicate; conjunctions; fragments and run-ons; punctuation)
  • Idioms (prepositions, correlatives, and forms of comparison)
  • Modifiers (dangling and misplaced)
  • Verb tenses and moods
  • Verbals (noun and adjective uses of verbs)

Command over grammar is the most vital skill to have for sentence correction questions, but grammar is still just one part of what you’re being tested on: you will encounter sentences that are grammatically correct but are still awkward or redundant and need to be reworded for more clarity and concision.

 

A solid command of grammar is handy for the sentence correction questions!
A solid command of grammar is handy for the sentence correction questions!

 

4 Tips for Key GMAT Subjects to Study

Now that we’ve gone over the topics and question formats covered on the GMAT, let’s dive into the key subjects to study for each section.

 

Study Persuasive Writing for the Analytical Writing Assessment

As stated above, though a formal understanding of logic is not required for the AWA, you can see that it’s important to be well-versed in persuasive writing and to understand concepts like counterargument and rebuttal, faulty reasoning, and claims and evidence.

Here’s a useful one-pager on Toulmin Analysis to get you started. Toulmin Analysis is a great academic entry point into basic tenets of creating and rebutting an argument, particularly if you’re unfamiliar with foundational terms like “counterargument” and “rebuttal.” You should also read mba.com’s sample top-scoring AWA essay and list of real AWA prompts to familiarize yourself with the kinds of arguments presented and the responses that will score highly.

 

Study Graphs and Charts for Integrated Reasoning

When you study for the GMAT Quant and Verbal sections, you’re already learning the basic academic knowledge needed for IR section. In addition to those subjects, you should make sure to study up on the fundamentals of graphs and charts, tables, and spreadsheets so that you’re not surprised by any data presentation format on the test.

 

Elephants never forget their grammar rules!
Elephants never forget their number properties!

 

Memorize the Fundamental Rules, Values, and Properties for Quant

The key to mastering Quant is to memorize the key equation rules, values, properties and shortcut within the subject areas listed in the Quant subsection above, because you’ll need to do quick mental math and you will not have access to a calculator.

Above all, the absolute most important math topics to know inside and out are:

  • Number properties (especially the properties of integers)
  • Factors and multiples
  • Fractions, decimals and ratios
  • Statistics
  • Properties of sets
  • Percentages
  • Rates
  • Exponents and powers
  • Geometric properties of basic shapes and coordinate geometry
  • Systems of equations/isolating and solving for a variable
  • Inequalities

 

Read and Memorize GMAT Grammar Rules for Verbal

The most important method for studying for the Verbal is to read. While there is no outside knowledge expected in any of the key subject areas that you’ll see on the Verbal section, you should get in the habit of reading one article a day from the kinds of magazines and periodicals from which they source arguments and passages. This strategy has a variety of benefits, including familiarizing yourself with the subjects, increasing your critical reading skills, and honing your intuitive grasp of grammar and syntax. Some great sources we recommend to get you started are National Geographic, The Economist, and Scientific American.

For sentence correction questions, the grammar rules that you’ll be tested on are a bit particular, so you should just go ahead and memorize them. Here is a comprehensive PDF of all of the grammar you’ll need to know for the GMAT for exactly that purpose (coming soon).

 

What’s Next?

The next step is to learn how each section on the GMAT is scored to complete your foundational knowledge of the exam. If you still have questions about GMAT subjects or other fundamentals of the test, check out our guides to what the GMAT actually is, how hard it is, and how much time you should budget to prepare for it too.

Unsure which test to take for business school? We can help you compare and contrast the GMAT with the GRE as well.

Happy studying!

The post What Is on the GMAT? Expert Guide to GMAT Subjects appeared first on Online GMAT Prep Blog by PrepScholar.

]]>